You are on page 1of 67

American Board of Family Practice

2002 IN-TRAINING EXAMINATION CRITIQUE BOOK

This book contains the correct answers to all the test items in the 2002 In-Training Examination. In addition, each item is accompanied by a rationale for the correct answer. Bibliographic references are included at the end of each critique to facilitate any further study you may wish to do in a particular area.

Copyright 2002 The American Board of Family Practice. All rights reserved.

BOOK I
Item 1 ANSWER: A

Subluxation of the radial head (nursemaids elbow) is a common childhood orthopedic problem. There is conflicting information in various textbooks on the proper technique to reduce the subluxed radial head. However, a study comparing hyperpronation to supination/flexion found that hyperpronation had a higher success rate. If a fracture is diagnosed, then a sling would be helpful. Neither a wrist splint nor a long arm cast is helpful in this situation.
Ref: Macias CG, Bothner J, Wiebe R: A comparison of supination/flexion to hyperpronation in the reduction of radial head subluxations. Pediatrics 1998;102(1):e10

Item 2 ANSWER: E

Pyelonephritis is the most common medical complication of pregnancy. The diagnosis is usually straightforward, as in this case. Since the patient is quite ill, treatment is best undertaken in the hospital, at least until the patient is stabilized and cultures are available. Ampicillin is widely used as the agent of first choice, but because of variable drug resistance some studies suggest adding an aminoglycoside for a woman who is seriously ill. Alternatively, an extended-spectrum penicillin or a third-generation cephalosporin may be used. Sulfonamides are contraindicated late in pregnancy because they may increase the incidence of kernicterus. Tetracyclines are contraindicated because administration late in pregnancy may lead to discoloration of the childs deciduous teeth. Nitrofurantoin may induce hemolysis in women who are deficient in G-6-PD, which includes approximately 2% of African-American women. The safety of levofloxacin in pregnancy has not been established, and it should not be used unless the potential benefit outweighs the risk.
Ref: Cunningham FG, Gant NF, Leveno KJ, et al: Williams Obstetrics, ed 21. McGraw-Hill, 2001, pp 1255-1258.

Item 3 ANSWER: D

A thrombosed external hemorrhoid is described as the sudden development of a painful, tender perirectal lump. Because there is somatic innervation, the pain is intense, and increases with edema. Treatment involves excision of the acutely thrombosed tissue under local anesthesia, mild pain medication, and sitz baths. It is inappropriate to use procedures that would increase the pain, such as banding or cryotherapy. Total hemorrhoidectomy is inappropriate and unnecessary.
Ref: Schwartz SI (ed): Principles of Surgery, ed 7. McGraw-Hill, 1999, pp 1297-1298.

Item 4 ANSWER: B

Guttate psoriasis occurs predominantly in children and is characterized by small oval lesions on the trunk and proximal extremities. The onset frequently follows a streptococcal respiratory infection. Scarlet fever usually accompanies streptococcal pharyngitis and the rash is red, punctate, and often felt more readily than seen. Atopic dermatitis is characterized by chronic dry skin which is pruritic. Scabies is not related to streptococcal infection and is a papular, itchy rash seen on the finger webs, axillae, belt line, and genital areas. Erythema marginatum is a manifestation of rheumatic fever and is a nonspecific macular lesion of the trunk with central blanching that appears serpiginous.
Ref: Behrman RE, Kliegman RM, Jenson HB (eds): Nelson Textbook of Pediatrics , ed 16. WB Saunders Co, 2000, pp 2001-2002.

Item 5 ANSWER: B

While rarely life threatening, an acute dystonic reaction can be frightening and painful to the patient and confusing to the treating physician who may be unaware of what medications the patient is taking. Dystonia can be caused by any agent that blocks dopamine, including prochlorperazine, metoclopramide, and typical neuroleptic agents such as haloperidol. The acute treatment of choice is diphenhydramine or benztropine.
Ref: Sadock BJ, Sadock VA (eds): Kaplan & Sadocks Comprehensive Textbook of Psychiatry, ed 7. Lippincott Williams & Wilkins, 2000, p 2280.

Item 6 ANSWER: D

According to the most recent guidelines from the Centers for Disease Control, pneumococcal vaccine should be given to ALL persons 65 or over, including both previously unvaccinated persons and those who received the vaccine before age 65 but were vaccinated over 5 years ago. This patient was age 62 when he received the vaccine, indicating he should be revaccinated at the time of the health maintenance visit. Data from a single epidemiologic study suggest that the vaccine may provide protection for at least 9 years after receipt of the initial dose; however, data to support the need for subsequent doses of pneumococcal vaccine are not available, and because there is insufficient data concerning the safety of pneumococcal vaccine when administered three or more times, revaccination following a second dose is not routinely recommended.
Ref: Prevention of pneumococcal disease: Recommendations of the Advisory Committee on Immunization Practices (ACIP). MMWR 1997;46(RR-8):10-15.

Item 7 ANSWER: A

This patient has rheumatoid arthritis (RA) by symptoms and physical findings. A positive latex fixation test for rheumatoid factor is not necessary for the diagnosis. A negative rheumatoid factor does not exclude RA, and a positive rheumatoid factor is not specific. Rheumatoid factor is found in the serum of approximately 85% of adult patients with RA; in subjects without RA, the incidence of positive rheumatoid factor is 1%5% and increases with age. The ANA test is positive in at least 95% of patients with systemic lupus erythematosus (SLE), but in only about 35% of patients with RA. Elevation of the erythrocyte sedimentation rate is seen in many patients with RA, and the degree of elevation roughly parallels disease activity. A mean of 6 months after the onset of Lyme disease, 60% of patients in the U.S. have brief attacks of asymmetric, oligoarticular arthritis, primarily in the large joints and especially in the knee.
Ref: Klippel JH (ed): Primer on the Rheumatic Diseases, ed 12. Arthritis Foundation, 2001, pp 209-225.

Item 8 ANSWER: B

Thrombolysis is now an approved treatment for acute stroke. The critical time frame is 3 hours after the onset of the deficit. Beyond that time span, the use of thrombolytic agents is contraindicated. Advanced age per se is not a contraindication to thrombolytic therapy. Contraindications include blood glucose levels <50 mg/dL or >400 mg/dL, resolving transient ischemic attack, and hemorrhage visible on a CT scan.
Ref: Rakel RE, Bope ET (eds): Conns Current Therapy 2002. WB Saunders Co, 2002, pp 874-876.

Item 9 ANSWER: A

Illegal drug use is currently the leading cause of new cases of hepatitis C. It is estimated that 60% of new cases of hepatitis C in the United States are due to injection drug use. Intranasal cocaine use has been associated with hepatitis C, but its importance as a route of transmission is controversial and it occurs at a much lower frequency, if it all. Sexual transmission of hepatitis C is a less frequent cause of initial infection. Hepatitis B and HIV are transmitted more efficiently by sexual contact. Men who engage in homosexual intercourse have rates of hepatitis C similar to those of heterosexuals who engage in high-risk sexual practices. Among partners who are hepatitis Cpositive, male-to-female transmission seems to be more efficient. Maternal-child perinatal transmission rates are thought to be less than 6%. Breastfeeding is not thought to be a risk. Occupational transmission is infrequent. No significant household or day-care risk is thought to exist in the absence of blood exposure.
Ref: Williams I: Epidemiology of hepatitis C in the United States. Am J Med 1999;107(6B):2S-9S.

Item 10 ANSWER: B

Paroxetine has emerged as the drug of choice for treatment of social phobias. Also known as social anxiety disorder, social phobias characteristically cause fear of situations that may lead to embarrassing scrutiny. Patients learn to avoid situations where they feel others may notice them, such as church gatherings, classroom settings, and other group events. Lithium and haloperidol are used for more severe psychiatric disturbances, while alprazolam would be best reserved for secondary use because of the possibility of dependency. Trazodone is approved for treatment of depression, but its strong sedative properties make it inappropriate in this scenario.
Ref: Brown CS: Depression and anxiety disorders. Obstet Gynecol Clin North Am 2001;28(2):241-268. 2) Braunwald E, Fauci AS, Kasper DL, et al (eds): Harrisons Principles of Internal Medicine, ed 15. McGraw-Hill, 2001, pp 2543-2544.

Item 11 ANSWER: C

In most patients with heavy gastrointestinal bleeding, localizing the bleeding site, rather than diagnosing the cause of the bleeding, is the most important task. A lower GI series is usually nondiagnostic during heavy, active bleeding. A small-bowel radiograph may be helpful after the active bleeding has stopped, but not during the acute phase of the bleeding. A blood pool scan allows repeated scanning over a prolonged period of time, with the goal of permitting enough accumulation of the isotope to direct the arteriographer to the most likely source of the bleeding. If the scan is negative, arteriography would also be unlikely to reveal the active source of bleeding. It is also a more invasive procedure. Exploratory laparotomy may be indicated if a blood pool scan or an arteriogram is nondiagnostic and the patient continues to bleed heavily.
Ref: Schwartz SI (ed): Principles of Surgery, ed 7. McGraw-Hill, 1999, pp 1065-1066. 2) Braunwald E, Fauci AS, Kasper DL, et al (eds): Harrisons Principles of Internal Medicine, ed 15. McGraw-Hill, 2001, pp 253-254.

Item 12 ANSWER: B

Ninety percent of early and immediate postpartum hemorrhage is due to failure of the uterus to contract satisfactorily (uterine atony). Other less frequent causes are lacerations of the cervix, vagina, or perineum; hematomas, usually located near lacerations or episiotomy repairs; and uterine rupture, either spontaneous or iatrogenic. All of these occur in the immediate postpartum period. Delayed hemorrhage, occurring beyond the first 24 hours after delivery, is usually caused by retained placental fragments. Interestingly, placenta accreta is among the most common causes of postpartum hemorrhage necessitating hysterectomy.
Ref: Ripley DL: Uterine emergencies: Atony, inversion, and rupture. Obstet Gynecol Clin North Am 1999;26(3):419-434. 2) Cunningham FG, Gant NF, Leveno KJ, et al: Williams Obstetrics, ed 21. McGraw-Hill, 2001, p 635.

Item 13 ANSWER: D

Metformin should be withheld before and after radiographic procedures with contrast, due to its interaction with iodinated contrast materials. This interaction may cause impaired renal function or lactic acidosis. The other drugs listed do not carry this risk.
Ref: Drug Facts and Comparisons, 2001 Edition. Facts and Comparisons, 2001, pp 308-311.

Item 14 ANSWER: D

The individual described in this case has symptomatic hyponatremia. Headache, mental confusion, nausea, and malaise are common. Seizures, stupor, and coma generally do not occur until sodium concentrations fall below 120 mEq/L. The presence of significant peripheral edema in this patient indicates extracellular fluid volume expansion, and his serum osmolality is low. In this situation, hyponatremia is usually a manifestation of an edematous state, such as hepatic cirrhosis, congestive heart failure, or the nephrotic syndrome. Although these patients have increased extracellular fluid, their intravascular fluid is depleted, and their bodys attempt to conserve sodium at the level of the kidney produces urine with a sodium concentration <20 mEq/L. They have appropriately increased arginine vasopressin (AVP) levels, resulting in a urine osmolality that is less than maximally dilute and often >100 mOsm/kg H 2 O. Patients with the syndrome of inappropriate antidiuretic hormone (SIADH) have normal volume status and urine sodium levels which are typically >20 mEq/L. Patients with primary polydipsia often have an underlying psychiatric disorder. They have normal volume status, and produce large volumes of very dilute urine (<50 mOsm/kg H2 O). Patients with adrenal insufficiency typically have normal volume status, but may be dehydrated. Patients with salt-wasting nephropathy are typically dehydrated despite producing urine with a sodium concentration >20 mEq/L.
Ref: Braunwald E, Fauci AS, Kasper DL, et al (eds): Harrisons Principles of Internal Medicine, ed 15. McGraw-Hill, 2001, pp 274-276.

Item 15 ANSWER: D

Larva migrans is transmitted by skin contact with soil contaminated with hookworm larvae from dog and cat feces in tropical and subtropical areas. Wearing shoes and sitting on a towel rather than directly on the ground are protective measures.
Ref: Kitchen LW: Case studies in international travelers. Am Fam Physician 1999;60(2):471-474.

Item 16 ANSWER: B

For long-term therapy, $-adrenergic antagonist drugs provide the most effective control of heart rate in atrial fibrillation, both at rest and during exercise. Although calcium channel blockers also lower heart rate both at rest and with exercise, they are not as effective as $-blockers. Digitalis is primarily effective in controlling the heart rate at rest, and often does not adequately control heart rate with exercise. The Class 1 antiarrhythmics are most useful in maintaining sinus rhythm and, in fact, may paradoxically increase heart rate.
Ref: Lampert R, Ezekowitz MD: Management of arrhythmias. Clin Geriatr Med 2000;16(3):593-618.

Item 17 ANSWER: D

In the late 1980s human papillomavirus (HPV) was linked to cervical cancer and clinicians were advised to aggressively treat all patients with suspected HPV disease, including those with genital warts. More recent data indicate clearly that cervical cancer and genital warts result from different HPV types and do not appear to be linked. Specifically, external genital warts usually contain HPV types 6 or 11, and cervical cancer is associated with HPV types 16, 18, 31, 33, and others. In the 1970s, it was demonstrated that a single sexual contact with a person infected with external genital warts carries about a 60% chance of transmission. A history of multiple partners is the most important risk factor for acquiring HPV infection. Because it is not possible to culture papillomavirus in cells, alternative methods are used to detect genital HPV expression: inspection with the unaided eye or with hand-held lens magnification, colposcopy, cytologic smears or biopsies, and viral DNA detection methods.
Ref: Verdon ME: Issues in the management of human papillomavirus genital disease. Am Fam Physician 1997;55(5):1813-1822.

Item 18 ANSWER: A

The 30 oblique position is well tolerated by elderly patients and can be used in almost any setting. The once common 90 lateral position leads to trochanteric ulcers. The supine position contributes to the already present sacral decubitus, and thus should be avoided, even as part of a regular turning schedule or pressure-reducing measures.
Ref: Evans JG, Williams TF, Beattie BL, et al (eds): Oxford Textbook of Geriatric Medicine. Oxford University Press, 2000, pp 105, 106, 108.

Item 19 ANSWER: C

There is no known vegetable source for vitamin B12 . If the diet contains milk and eggs, however, the requirement will be satisfied. Vitamin C is present in high concentrations in fruits and vegetables. Vitamin A is found in green, leafy vegetables. Thiamine is present in grains, especially the outer coatings.
Ref: Behrman RE, Kliegman RM, Jenson HB (eds): Nelson Textbook of Pediatrics , ed 16. WB Saunders Co, 2000, p 168.

Item 20 ANSWER: D

College freshmen, especially those who live in dormitories, are at a modestly increased risk for meningococcal disease compared with other persons of the same age, and vaccination with the currently available quadrivalent meningococcal polysaccharide vaccine will decrease their risk. The Advisory Committee on Immunization Practices of the Centers for Disease Control and Prevention recommends that incoming and current college freshmen, and their parents, be informed about meningococcal disease and the benefits of vaccination. This is particularly true for those who live in dormitories and residence halls. In this case, the patient has received vaccinations against measles, polio, pertussis, and tetanus at the recommended times, and booster vaccinations are not indicated now.
Ref: Meningococcal disease and college students: Recommendations of the Advisory Committee on Immunization Practices(ACIP). MMWR 2000;49(RR-7):13-20.

Item 21 ANSWER: E

A postmenopausal woman with an ovarian cyst >3 cm in size associated with elevated CA-125, especially a level >100 U/mL, should be referred to a surgeon for evaluation. Other benign conditions could cause elevation of CA-125, but usually to levels <100 U/mL. The ultrasonography and CA-125 could be repeated periodically if the cyst were <3 cm and the CA-125 level were normal. Repeat clinical examination would be inadequate, and reexamination following treatment with estrogen plus progestin would add no useful information. All of the other options would only delay the surgical evaluation, which is essential in this situation.
Ref: Drake J: Diagnosis and management of the adnexal mass. Am Fam Physician 1998;57(10):2471-2476, 2479-2480.

Item 22 ANSWER: B

Lower urinary tract obstruction in a newborn or young boy should be assumed to be caused by posterior urethral valves until proven otherwise. These valves consist of folds of mucosa obstructing the urethra at the prostatic level. The diagnosis can be confirmed by voiding cystourethrography or by endoscopy of the urethra. The prognosis depends on the extent of renal damage at the time of diagnosis.
Ref: Rudolph AM (ed): Rudolphs Pediatrics , ed 20. Appleton & Lange, 1996, pp 1397-1398. 2) Behrman RE, Kliegman RM, Jenson HB (eds): Nelson Textbook of Pediatrics , ed 16. WB Saunders Co, 2000, pp 1636-1637.

Item 23 ANSWER: C

Getting up at the same time every day stabilizes the sleep-wake schedule and improves the restfulness of actual sleep in bed. Insomnia is more prevalent among the elderly and is associated with depression. Daytime drowsiness, excessive snoring, and confusion when awakened are associated with sleep apnea. Hypnotics should be prescribed only for short-term use.
Ref: Kupfer DJ, Reynolds CF III: Management of insomnia. N Engl J Med 1997;336(5):341-346.

Item 24 ANSWER: D

Routine coagulation tests such as prothrombin time and partial thromboplastin time are insensitive measurements of Lovenox activity. Antifactor Xa can be measured in patients with renal failure to monitor anticoagulation effects.
Ref: Hardman JG, Limbird LE, Gilman AG (eds): Goodman & Gilmans The Pharmacological Basis of Therapeutics, ed 10. McGraw-Hill, 2001, p 1524.

Item 25 ANSWER: B

When caloric intake is greater than energy expenditure, obesity results. The major components of caloric expenditure are the basal metabolic rate, dietary thermogenesis, and physical activity. The basal metabolic rate (the amount of energy required to maintain metabolic homeostasis) accounts for 60%70% of caloric expenditure. Dietary thermogenesis (the energy required for digestion and metabolism of food) accounts for 5%10% of caloric expenditure and depends upon the amount and type of food eaten and, to an extent, on the degree of obesity. Physical activity accounts for about 25%35% of caloric expenditure in the average person.
Ref: Braunwald E, Fauci AS, Kasper DL, et al (eds): Harrisons Principles of Internal Medicine, ed 15. McGraw-Hill, 2001, pp 479-480.

Item 26 ANSWER: C

The most significant anomaly associated with urinary tract infections (UTIs) in children is vesicoureteral reflux, which occurs in 30%50% of children with UTIs. When screening a child for reflux, the initial test should be voiding cystourethrography. Although renal ultrasonography is less invasive, findings are normal in 50%75% of patients with reflux. A DMSA renal flow scan is the best study for detecting renal scarring, but will not detect reflux in children who have not yet developed scarring. Nuclear cystography is as sensitive for detecting reflux as a standard VCUG, but grading of reflux is less precise and this test will not detect associated bladder abnormalities.
Ref: Ross JH, Kay R: Pediatric urinary tract infection and reflux. Am Fam Physician 1999;59(6):1472-1478, 1485-1486.

Item 27 ANSWER: B

Emergency postcoital contraception, commonly called morning after protection (MAP) is intended to be one-time, emergency protection, not regular contraception. Combination estrogen-progesterone pills can be given in a form that contains 50 mg ethinyl estradiol and 0.5 mg norgestrel. Two pills are given immediately and two more 12 hours later. This method is considered effective if given within 72 hours of unprotected intercourse. Levonorgestrel in a dose of 0.75 mg (also called Plan B) is given in two doses 12 hours apart, also within 72 hours. This dose is equivalent to 10 pills of the levonorgestrel progestin-only minipill. Plan B may be better tolerated, but is not as readily available in family physicians offices. Endometrial aspiration is not 100% effective, and if it were used for contraception many unnecessary procedures would be done. Diethylstilbestrol was the choice for MAP for a few years but is rarely used now; in addition, the correct dosage is 2550 mg/day for 5 days. Nausea and menstrual irregularity were common side effects, and both physicians and patients were uneasy about using a potential teratogen. Intrauterine devices are not generally recommended in nulliparous women and should be used only in women who wish to use the device on a continual basis and who are not at high risk for sexually transmitted diseases.
Ref: Speroff L, Glass RH, Kase NG: Clinical Gynecologic Endocrinology and Infertility, ed 6. Lippincott Williams & Wilkins, 1999, pp 929-930.

Item 28 ANSWER: E

Routine ultrasonography at around 1822 weeks gestation has become the standard of care in many communities. Acceptance is based on many factors, including patient preference, medical-legal pressure, and the perceived benefit by physicians. However, rigorous testing has found little scientific benefit for, or harm from, routine screening ultrasonography.
Ref: Ewigman BG, Crane JP, Frigoletto FD, et al: Effect of prenatal ultrasound screening on perinatal outcome. N Engl J Med 1993;329(12):821-827. 2) Neilson JP: Ultrasound for fetal assessment in early pregnancy. Cochrane Database Syst Rev 2000;(2):CD000182.

Item 29 ANSWER: A

Once the diagnosis of anorexia nervosa is confirmed, the initial goal of treatment is to prevent death by starvation. Depression, a common finding in anorexia nervosa, is usually alleviated with nourishment. In cases that are refractory to proper nutrition, an antidepressant may be helpful. Psychotherapy, using a combination of behavioral and cognitive techniques, is an important adjunctive therapy. It begins when the diagnosis is established and continues after the patient has returned to normal weight. Family therapy is also recommended in younger patients. It facilitates recovery in the individual by addressing problems in the family environment and also often continues for years after the patients return to normal weight.
Ref: Hobbs WL, Johnson CA: Anorexia nervosa: An overview. Am Fam Physician 1996;54(4):1273-1279, 1284-1286. 2) Kaplan HI, Sadock BJ: Kaplan & Sadocks Synopsis of Psychiatry: Behavioral Sciences/Clinical Psychiatry, ed 8. Lippincott Williams & Wilkins, 1998, pp 725-726.

Item 30 ANSWER: E

Epididymitis is an inflammation of the epididymis due to various infectious agents or to local trauma. Pain may develop over a day or two, or even more gradually. In men under the age of 35 who are sexually active, the most common etiologic organisms are Chlamydia trachomatis and Neisseria gonorrhoeae. In men over 35, infectious epididymitis is usually nonspecific and is caused by coliform bacteria or Pseudomonas species. The preferred treatment is ofloxacin, 400 mg orally twice a day for 10 days. Alternative therapy is a single dose of ceftriaxone, 250 mg, plus doxycycline, 100 mg twice a day for 10 days.
Ref: Hausam R, Killeen I, Osborn C: Sexually Transmitted Disease. AAFP Home Study Self-Assessment monograph series, 1997, no 215, p 23. 2) Mandell GL, Bennett JE, Dolin R (eds): Mandell, Douglas, and Bennetts Principles and Practice of Infectious Diseases, ed 5. Churchill Livingstone, 2000, p 1247.

Item 31 ANSWER: B

$-Blockers with intrinsic sympathomimetic activity (ISA) are less beneficial in reducing mortality post myocardial infarction, and for this reason are not recommended for ischemic heart disease. They have a potential advantage in only one clinical situation. Since they tend to lower heart rates less, they may be beneficial in patients with symptomatic bradycardia while taking other $-blockers. All $-blockers should be used cautiously in patients with diabetes or asthma. Only sotalol, which delays ventricular depolarization, has been shown to be effective for maintenance of sinus rhythm in patients with chronic atrial fibrillation.
Ref: Which beta-blocker? Med Lett Drugs Ther 2001;43:9-12.

10

Item 32 ANSWER: A

Clustered circular lesions with accompanying dysesthesia in a dermatome are characteristic of herpes zoster, which may occur after a stressful event or infection in both children and adults. For herpes zoster to occur there must be a previous primary varicella infection or immunization. Herpes zoster is less likely to be associated with significant postherpetic neuralgia in children than in adults.
Ref: Behrman RE, Kliegman RM, Jenson HB (eds): Nelson Textbook of Pediatrics , ed 16. WB Saunders Co, 2000, pp 973-977.

Item 33 ANSWER: E

Fever, headache, and myalgia must be recognized as the classic triad of Rocky Mountain spotted fever in all patients who might have been bitten by a tick, as commonly occurs in campers. This patient exhibits a classic picture of the disease, which often has a virulent 3-week course even when treated with tetracycline or chloramphenicol. A 10% mortality rate has been noted for this illness, which is not uncommon in the southeastern United States. Lyme disease is also the result of tick transmission, but is otherwise unlike Rocky Mountain spotted fever; gonococcemia is associated with monoarticular pain and a papular rash; systemic lupus erythematosus and immune complex diseases do not produce the clinical picture seen in this patient.
Ref: Goldman L, Bennett JC (eds): Cecil Textbook of Medicine, ed 21. WB Saunders Co, 2000, pp 1771-1773.

Item 34 ANSWER: C

Chronic radiation proctitis develops months to years after radiation and is characterized by pain with defecation, diarrhea, and sometimes rectal bleeding. On colonoscopy, the mucosa is pale and friable with telangiectases which are sometimes large, multiple, and serpiginous.
Ref: Schwartz SI: Principles of Surgery, ed 7. McGraw-Hill, 1999, pp 1309-1310. 2) Bonis PAL, Nostrant TT: Diagnosis and treatment of chronic radiation proctitis. UpToDate 2000;8(2).

Item 35 ANSWER: B

Erythema infectiosum is characterized by a prodromal illness usually consisting of malaise, pharyngitis, and low-grade fever, followed by the appearance of a slapped-cheek rash. The cause of erythema infectiosum has been identified as parvovirus B19.
Ref: Rudolph AM (ed): Rudolphs Pediatrics , ed 20. Appleton & Lange, 1996, pp 668-669.

11

Item 36 ANSWER: C

In a study of patients with low to moderate caffeine intake, discontinuing caffeine resulted in moderate to severe headache in 52%. About 10% had depression or anxiety, and less than 10% had fatigue or flu-like symptoms.
Ref: Silverman K, Evans SM, Strain EC, et al: Withdrawal syndrome after the double-blind cessation of caffeine consumption. N Engl J Med 1992;327(16):1109-1114. 2) Tierney LM Jr, McPhee SJ, Papadakis MA (eds): Current Medical Diagnosis & Treatment, ed 38. Appleton & Lange, 1999, p 1040.

Item 37 ANSWER: D

This patient has typical symptoms and laboratory findings of multiple myeloma, which accounts for 1% of all malignant diseases and has a mean age at diagnosis of 61 years. The diagnosis is confirmed by a bone marrow examination showing >10% plasma cells in the marrow. The serum level of M-protein is typically >3 g/dL. A bone scan is inferior to conventional radiography and should not be used. Tamoxifen is indicated for the treatment of breast cancer, which is unlikely given the physical and laboratory findings in this case.
Ref: Braunwald E, Fauci AS, Kasper DL, et al (eds): Harrisons Principles of Internal Medicine, ed 15. McGraw-Hill, 2001, pp 728-730.

Item 38 ANSWER: B

Running injuries are primarily caused by overuse due to training errors. Runners should be instructed to increase their mileage gradually, in increments of 10% or less each week. A stress fracture causes localized tenderness and swelling in superficial bones. Pain is reproduced by jumping on the affected leg. Plantar fasciitis causes burning pain in the heel and there is tenderness of the plantar fascia where it inserts onto the medial tubercle of the calcaneus.
Ref: Ballas MT, Tytko J, Cookson D: Common overuse running injuries: Diagnosis and management. Am Fam Physician 1997;55(7):2473-2480.

Item 39 ANSWER: A

Cardiac rehabilitation programs are safe and effective. Rates of coronary events in rehabilitation settings are very low. Cardiac rehabilitation consistently improves exercise capacity and has favorable effects on coronary risk factors, even without nutritional counseling. A baseline exercise stress test prior to starting cardiac rehabilitation is necessary to screen for exertional ischemia or arrhythmias, and serves as a basis for prescribing an exercise regimen. The exercise prescription is individualized based on results of the exercise stress test, the age of the patient, and the patients clinical status.

12

Ref:

Ades PA: Cardiac rehabilitation and secondary prevention of coronary heart disease. N Engl J Med 2001;345(12):892-902.

Item 40 ANSWER: E

Gastroesophageal reflux is a common cause of wheezing in infants. At 5 months of age, most infants no longer spit up several times a day, and this is a major clue that the wheezing may be from the reflux. Also, there is no family history of asthma and the wheezing is not related to infections. Cystic fibrosis is more likely to present with recurrent infections and failure to thrive than with intermittent wheezing.
Ref: Hart JJ: Pediatric gastroesophageal reflux. Am Fam Physician 1996;54(8):2463-2472. 2) Behrman RE, Kliegman RM, Jenson HB (eds): Nelson Textbook of Pediatrics , ed 16. WB Saunders Co, 2000, pp 1125-1126.

Item 41 ANSWER: B

Vitamin D deficiency is being recognized more frequently among the elderly, especially in the nursing home or other settings where inactivity, nutritional deficiency, and lack of sunlight exposure combine to increase the propensity for deficiency. Vitamin D deficiency has been associated with abnormalities in bone metabolism, primarily osteomalacia. Clinically this can present as muscle weakness, limb pain, and impaired physical function. Bleeding gums and fatigue may be presenting symptoms of scurvy (vitamin C deficiency). High-output cardiac failure with vasodilation, dermatitis, and neuropathy is the clinical presentation of thiamine deficiency. The combination of diarrhea, scaly dermatitis, and dementia is one of the clinical presentations of niacin deficiency.
Ref: Gloth FM III, Tobin JD: Vitamin D deficiency in older people. J Am Geriatr Soc 1995;43(7):822-828. 2) Hazzard WR, Blass JP, Ettinger WH Jr, et al (eds): Principles of Geriatric Medicine and Gerontology, ed 4. McGraw-Hill, 1999, pp 94, 1080-1083.

Item 42 ANSWER: D

In a pregnancy complicated by bleeding at less than 8 weeks gestation, it is often difficult to determine whether there is a viable intrauterine pregnancy, a nonviable intrauterine pregnancy, or an ectopic pregnancy. Vaginal ultrasonography can be used to detect intrauterine gestation as early as 1 week after conception if the $-hCG level is >1500 mIU/mL. An empty uterus with a $-hCG level >1500 mIU/mL is highly suggestive of ectopic pregnancy. When the $-hCG is <1500 mIU/mL and vaginal ultrasonography is nondiagnostic, the $-hCG should be repeated in 23 days. Failure of the $-hCG to double in 23 days suggests a blighted ovum or ectopic pregnancy. The efficacy of progesterone in early pregnancy has not been proven, and there is some potential risk (virilization of the female fetus). An ectopic pregnancy is possible in this situation, but would be more likely if the hCG were higher in the presence of an empty uterus, or if there were an adnexal mass on physical examination or ultrasonography.
Ref: Cunningham FG, Gant NF, Leveno KJ, et al: Williams Obstetrics, ed 21. McGraw-Hill, 2001, pp 892-893.

13

Item 43 ANSWER: C

The only option listed that is contraindicated for circumcision is lidocaine with epinephrine. Epinephrine should NEVER be used in a local anesthetic for penile block because the penile artery is an end artery, and vasospasm can lead to necrosis of the penile tip.
Ref: Wilder RT: Local anesthetics for the pediatric patient. Pediatr Clin North Am 2000;47(3):545-558.

Item 44 ANSWER: E

An astonishingly high prevalence of microscopic foci of well-differentiated adenocarcinoma has been found at autopsy in serial sections of prostate glands considered to be normal from men over the age of 50. The incidence of such tumors nearly doubles with each decade of age, starting at 10% for men in their 50s and rising to 70% for men in their 80s.
Ref: Naitoh J, Zeiner RL, Dekernion JB: Diagnosis and treatment of prostate cancer. Am Fam Physician 1998;57(7):1531-1539, 1541-1542, 1545-1547.

Item 45 ANSWER: B

There are many drugs that can induce a syndrome resembling systemic lupus erythematosus, but the most common offender is procainamide, followed by hydralazine. There is a genetic predisposition for this drug-induced lupus, determined by drug acetylation rates. Polyarthritis and pleuropericarditis occur in half of patients, but fortunately, CNS and renal involvement are rare. While all patients with this condition have positive ANAs and most have antibodies to histones, antibodies to double-stranded DNA and decreased complement levels are rare, which distinguishes drug-induced lupus from idiopathic lupus. The best initial management is to withdraw the drug, and most patients improve in a few weeks. For those with severe symptoms, a short course of corticosteroids is indicated. Once the offending drug is discontinued, symptoms seldom last beyond 6 months.
Ref: Braunwald E, Fauci AS, Kasper DL, et al (eds): Harrisons Principles of Internal Medicine, ed 15. McGraw-Hill, 2001, p 1926. 2) Klippel JH (ed): Primer on the Rheumatic Diseases, ed 12. Arthritis Foundation, 2001, p 345.

Item 46 ANSWER: B

Autonomic hyperreflexia is characterized by the sudden onset of headache and hypertension in a patient with a lesion above the T6 level. There may be associated bradycardia, sweating, dilated pupils, blurred vision, nasal stuffiness, flushing, or piloerection. It usually occurs several months after the injury and has

14

an incidence as high as 85% in quadriplegic patients. Frequently, it subsides within 3 years of injury, but it can recur at any time. Bowel and bladder distention are common causes. Hypertension is the major concern because of associated seizures and cerebral hemorrhage. Cluster headaches have a constant unilateral orbital localization. The pain is steady (non-throbbing) and lacrimation and rhinorrhea may be part of the syndrome. Sepsis is usually manifested by chills, fever, nausea, and vomiting. Common signs include tachycardia and hypotension rather than bradycardia and hypertension. Signs and symptoms of intracranial hemorrhage vary depending upon the site of the hemorrhage, but the unchanged neurologic status and the lack of a history of hypertension decrease the likelihood of this diagnosis. There are no neurologic findings or history which suggest progression of the patients lesion at C6.
Ref: Braunwald E, Fauci AS, Kasper DL, et al (eds): Harrisons Principles of Internal Medicine, ed 15. McGraw-Hill, 2001, pp 70, 78, 799, 2385, 2419.

Item 47 ANSWER: B

Rapid mood swings and speech that is typically pressured and difficult to interpret are characteristics of a manic episode. These features are not characteristic of the other psychiatric conditions listed.
Ref: American Psychiatric Association: Diagnostic and Statistical Manual of Mental Disorders , ed 4. American Psychiatric Association, 1994, pp 287, 328-330, 417-423, 645-654.

Item 48 ANSWER: D

Women older than 65 years of age who have low serum TSH levels, indicating physiologic hyperthyroidism, are at increased risk for new hip and vertebral fractures. Use of thyroid hormone itself does not increase the risk of fracture if TSH levels are normal.
Ref: Bauer DC, Ettinger B, Nevitt MC, et al: Risk for fracture in women with low serum levels of thyroid-stimulating hormone. Ann Intern Med 2001;134(7):561-568.

Item 49 ANSWER: A

In a study of varicella vaccine exposure during pregnancy, rates of congenital varicella syndrome and congenital abnormalities were calculated for seronegative women in a registry of women receiving vaccine during pregnancy. No cases of congenital varicella syndrome and no specific pattern of congenital abnormalities were identified among women receiving the vaccine.
Ref: Shields KE, Galil K, Seward J, et al: Varicella vaccine exposure during pregnancy: Data from the first 5 years of the pregnancy registry. Obstet Gynecol 2001;98(1):14-19.

15

Item 50 ANSWER: D

Serum albumin is sensitive to changes in nutritional status, although its long half-life (1721 days) means that return to normal is slow. A lymphocyte count provides a quick estimate of immune status and low counts may indicate a need for nutritional assessment. Iron studies are more likely to be abnormal as a result of gastrointestinal blood loss. Folate levels mostly reflect generic nutritional deficiency, rather than a specific deficiency. A low folate level is consistent with consumption of a limited variety and quantity of food. Electrolyte levels do not reflect nutrition.
Ref: Ham RJ: The signs and symptoms of poor nutritional status. Prim Care 1994;21(1):33-54. 2) Hazzard WR, Blass JP, Ettinger WH Jr, et al (eds): Principles of Geriatric Medicine and Gerontology, ed 4. McGraw-Hill, 1999, p 91.

Item 51 ANSWER: B

Ovarian cancer is the deadliest gynecologic malignancy in the U.S. The risk for this cancer increases with age, with the greatest incidence in postmenopausal women. Screening for early detection of ovarian malignancies has not proven effective, and its benefit is quite controversial even in patients with a known genetic risk. Thus, primary prevention may prove to be the most important method for reducing morbidity and mortality from this malignancy. The pathogenesis of ovarian cancer is unclear, but it is strongly believed that constant, uninterrupted ovulation may lead to an increased risk. This may explain why events such as pregnancy, breastfeeding, and oral contraceptive use are associated with a decreased risk of ovarian cancer, while the use of fertility drugs poses an increased risk. Postmenopausal use of hormone replacement therapy is also associated with an increased risk of developing ovarian cancer, although the effect is weak and the overall risk is low.
Ref: Sutton MY, Santoro N, Hearns RM: Gynecologic health and disease. Med Clin North Am 1998;82(2):223-247. 2) Ryan KJ, Berkowitz RS, Barbieri RL, et al (eds): Kistners Gynecology and Womens Health, ed 7. Mosby Inc, 1999, pp 166-167, 188. 3) Sherif K: Benefits and risks of oral contraceptives. Am J Obstet Gynecol 1999;180(6 Pt 2):S343-S348.

Item 52 ANSWER: A

IVP would be a high-risk procedure in this patient because of his elevated creatinine, diabetes, and recent contrast exposure. Contrast exposure for coronary arteriography is one possible cause of his renal failure. Sonography or spiral CT could be done without contrast to rule out an obstruction or stone. Bladder catheterization carries a risk of infection, but would be useful in ruling out bladder outlet obstruction in an elderly male. It might also prove therapeutic. Fractional excretion of sodium is a useful test for differentiating prerenal azotemia from acute tubular necrosis, and should be done prior to diuretic administration.
Ref: Howell JM: Emergency Medicine. WB Saunders Co, 1998, pp 833-838. 2) Tierney LM Jr, McPhee SJ, Papadakis MA (eds): Current Medical Diagnosis & Treatment 2000, ed 39. Appleton & Lange, 2000, pp 890-892.

16

Item 53 ANSWER: A

$-Natriuretic peptide (BNP) is a 32-amino acid polypeptide secreted from the cardiac ventricles in response to ventricular volume expansion and pressure overload. The major source of BNP is the cardiac ventricles, and because of its minimal presence in storage granules, its release is directly proportional to ventricular dysfunction. It is a simple and rapid test that reliably predicts the presence or absence of left ventricular dysfunction on an echocardiogram.
Ref: Krishnaswamy P, Lubien E, Clopton P, et al: Utility of $-natriuretic peptide levels in identifying patients with left ventricular systolic or diastolic function. Am J Med 2001;111(4):274-279.

Item 54 ANSWER: C

Bupivacaine has an expected duration of action of 24 hours. Procaine, lidocaine, and chloroprocaine have durations of action ranging from 15 minutes to 1 hour.
Ref: Townsend CM Jr: Sabiston Textbook of Surgery , ed 16. WB Saunders Co, 2001, pp 278-279. 2) Drug Facts and Comparisons, 2001 Edition. Facts and Comparisons, 2001, p 1017.

Item 55 ANSWER: E

Whereas many antibiotics temporarily suppress nasopharyngeal colonization by Haemophilus influenzae type b, only rifampin is effective in eradicating the organism. It should therefore be administered to all attendees and staff of a day-care facility in which two or more children have been diagnosed with disease caused by H. influenzae, regardless of previous immunization status. The patients should also receive rifampin before returning to the center. Prophylaxis after a single case is controversial.
Ref: Pickering LK (ed): 2000 Red Book: Report of the Committee on Infectious Diseases, ed 25. American Academy of Pediatrics, 2000, pp 264-266.

Item 56 ANSWER: C

In primary hyperparathyroidism, hypercalcemia is the result of excessive PTH secretion by one or more abnormal, enlarged parathyroid glands. Laboratory findings in most patients with primary hyperparathyroidism reflect the mild clinical presentation of the disorder. The serum calcium level is often 1 mg/dL or more above the upper limits of normal.

17

Bone radiographs may show the classic changes of subperiosteal bone resorption in the occasional patient with hyperparathyroidism, but in most cases they are normal or may show osteopenia. Osteocalcin is an osteoblast-specific protein. It is a marker of increased skeletal turnover, and it is usually not indicated clinically. The development of highly sensitive and specific assays for intact, largely active PTH has simplified the assessment of parathyroid activity. Bone densitometry is a test to determine the degree of osteoporosis.
Ref: Goldman L, Bennett JC (eds): Cecil Textbook of Medicine, ed 21. WB Saunders Co, 2000, p 1401.

Item 57 ANSWER: C

Although the significance of elevated triglycerides and a low HDL in low-risk patients is somewhat uncertain, in a high-risk patient such as a diabetic, improvement in these results will lower the risk of subsequent cardiac events. In diabetics, metformin and thiazolidinediones (e.g., rosiglitazone) are more likely to improve lipid levels than are sulfonylureas. Nicotinic acid is problematic in diabetics, as it tends to cause deterioration in glucose control. Fibrates are good choices for this patient because they will lower the triglyceride level and raise the HDL level. Exercise and weight loss are likely to be helpful as well. Cholestyramine will raise triglyceride levels.
Ref: Donahoo WT, Eckel RH: Evaluation, treatment, and implications of hypertriglyceridemia. Primary Care Case Reviews 2001;4(2):53-61. 2) Position Statement: Management of dyslipidemia in adults with diabetes. Diabetes Care 2002;25(suppl):574-577.

Item 58 ANSWER: B

As men age, there are normal changes in sexual functioning, including an increase in the time required for sexual arousal, decreased penile sensitivity (leading to an increased need for direct tactile sensation to obtain or maintain an erection), increased time between erections, and less forcefulness of ejaculation. Retrograde ejaculation and the inability to sustain an erection to ejaculation are not considered normal accompaniments of aging.
Ref: Beers MH, Berkow R (eds): The Merck Manual of Geriatrics, ed 3. Merck Research Laboratories, 2000, pp 1158, 1165-1170.

Item 59 ANSWER: B

The treatment of choice for occasional acute symptoms of asthma is an inhaled $2 -adrenergic agonist such as albuterol, terbutaline, or pirbuterol. However, acute symptoms that do not respond to $-agonists should be treated with a short course of systemic corticosteroids.

18

Theophylline has limited usefulness for treatment of acute symptoms in patients with intermittent asthma; it is a less potent bronchodilator than subcutaneous or inhaled adrenergic drugs, and therapeutic serum concentrations can cause transient adverse effects such as nausea and central nervous system stimulation in patients who have not been taking the drug continuously. Cromolyn can decrease airway hyperreactivity, but has no bronchodilating activity and is useful only for prophylaxis. Inhaled corticosteroids should be used for suppressing the symptoms of chronic persistent asthma. Oral $2 -selective agonists are less effective and have a slower onset of action than the same drugs given by inhalation.
Ref: Drugs for asthma. Med Lett Drugs Ther 2000;42:19-24.

Item 60 ANSWER: C

In the ideal setting, the accuracy of fine-needle aspiration may be over 90%. Clinical information is critical in interpreting the results of fine-needle aspirationmore so than with a tissue biopsy because of the more limited sampling. It is crucial to determine whether the findings on fine-needle aspiration explain the clinical findings. Although the report from the mammogram and the biopsy are not ominous, they do not explain the clinical findings in this case. Immediate repeat fine-needle aspiration or, even better, a tissue biopsy is indicated. Proceeding directly to therapy, whether surgery or irradiation, is inappropriate because the diagnosis is not clearly established. Likewise, any delay in establishing the diagnosis is not appropriate.
Ref: Lieu D: Fine-needle aspiration: Technique and smear preparation. Am Fam Physician 1997;55(3):839-846, 853-854.

Item 61 ANSWER: B

A helpful guideline for assessing normal growth in the very young infant is that birth weight should be regained by the 14th day.
Ref: Behrman RE, Kliegman RM, Jenson HB (eds): Nelson Textbook of Pediatrics , ed 16. WB Saunders Co, 2000, p 32.

Item 62 ANSWER: A

In normal pressure hydrocephalus a mild impairment of memory typically develops gradually over weeks or months, accompanied by mental and physical slowness. The condition progresses insidiously to severe dementia. Patients also develop an unsteady gait and urinary incontinence, but there are no signs of increased intracranial pressure.

19

In Alzheimers disease the brain very gradually atrophies. A disturbance in memory for recent events is usually the first symptom, along with some disorientation to time and place; otherwise, there are no symptoms for some period of time. Subacute sclerosing panencephalitis usually occurs in children and young adults between the ages of 4 and 20 years and is characterized by deterioration in behavior and work. The most characteristic neurologic sign is mild clonus. Multiple sclerosis is characteristically marked by recurrent attacks of demyelinization. The clinical picture is pleomorphic, but there are usually sufficient typical features of incoordination, paresthesias, and visual complaints. Mental changes may occur in the advanced stages of the disease. About two-thirds of those affected are between the ages of 20 and 40.
Ref: Humes HD (ed): Kelleys Textbook of Internal Medicine, ed 4. Lippincott Williams & Wilkins, 2000, pp 2919-2921, 2954-2955. 2) Victor M, Ropper AH: Adams and Victors Principles of Neurology, ed 7. McGraw-Hill, 2001, pp 663-665.

Item 63 ANSWER: A

Dual-energy x-ray absorptiometry (DEXA) of the hip and spine is the method of choice for assessment of bone mineral density. Quantitative CT is the most sensitive method, but results in substantially greater radiation exposure. Plain radiographs are not sensitive enough to diagnose osteoporosis until total bone density has decreased by 50%. The predictive value of DEXA scans and ultrasound examinations of peripheral bones in assessing fracture risk at the hip or vertebrae is not clear.
Ref: South-Paul JE: Osteoporosis: Part I. Evaluation and assessment. Am Fam Physician 2001;63(5):897-904, 908.

Item 64 ANSWER: A

Direct observation of the sun without an adequate filter, which often occurs during a solar eclipse, results in a specific type of radiation injury termed solar (eclipse) retinopathy. The lens system of the eye focuses the suns light onto a small spot on the macula, usually in one eye only, producing a thermal burn. The resulting retinal edema may clear spontaneously with minor functional loss, or it may cause significant tissue atrophy, leaving a defect seen with an ophthalmoscope as a macular hole. This macular injury produces a permanent central scotoma. Visual blurring and difficulty with light perception are reversible manifestations of the tissue hypoxia associated with carbon monoxide poisoning. Dust and other particulate matter can produce injury to the cornea and conjunctiva. These lesions are painful, usually prompting medical attention and appropriate treatment. While severe contusions to the globe and periorbital structures can produce retinal detachment, the clinical vignette does not support a boxing-type injury. Although potatoes have eyes, there is no other significant relationship between potato salad and ophthalmologic disease.
Ref: Vaughan D, Asbury T, Riordan-Eva P: General Ophthalmology, ed 15. Appleton & Lange, 1999, pp 186, 371.

20

Item 65 ANSWER: C

Initial management of hypercalcemia of malignancy calls for fluid replacement with normal saline to correct the volume depletion that is invariably present and to enhance renal calcium excretion. The use of loop diuretics such as furosemide should be restricted to patients in danger of fluid overload, since these drugs can aggravate volume depletion and are not very effective alone in promoting renal calcium excretion. Although intravenous pamidronate has become the mainstay for the hypercalcemia of malignancy, it is considered only after the hypercalcemic patient has been rendered euvolemic by saline repletion. The same is true for the other calcium-lowering agents listed.
Ref: Gucalp R, Theriault R, Gill I, et al: Treatment of cancer-associated hypercalcemia. Arch Intern Med 1994;154(17):1935-1944. 2) Mundy GR, Guise TA: Hypercalcemia of malignancy. Am J Med 1997;103(2):134-145.

Item 66 ANSWER: C

Causes of low back pain include vertebral disk herniation and spinal stenosis. Numbness and muscle weakness may be present in both. Pain in spinal stenosis is relieved by sitting and aggravated by standing. Pain from a herniated disk is aggravated by sitting and relieved by standing.
Ref: Wipf JE, Deyo RA: Low back pain. Med Clin North Am 1995;79(2):231-246. 2) Braunwald E, Fauci AS, Kasper DL, et al (eds): Harrisons Principles of Internal Medicine, ed 15. McGraw-Hill, 2001, pp 82-84.

Item 67 ANSWER: D

Considering relative risk reduction without also considering the absolute rate can distort the importance of a therapy. A useful way to assess the importance of a therapy is to determine the number-needed-to-treat for that therapy. To calculate this number, the percentage of absolute risk reduction of a particular therapy is divided into 100. In the case in question, the absolute risk reduction would be 0.5% (0.5.01). Thus, the number-needed-to-treat for the example cited would be 200 (100/0.5).
Ref: Hart RG, Benavente O: Stroke: Part I. A clinical update on prevention. Am Fam Physician 1999;59(9):2475-2482, 2485.

21

Item 68 ANSWER: D

The scenario described represents a case of shoulder dystocia. This complication cannot be reliably predicted prior to delivery, and all physicians performing deliveries must be familiar with its presentation and management. Overly vigorous traction of the infants head or neck in this situation may cause serious damage to the infant. Having an assistant apply moderate suprapubic pressure with gentle downward traction of the fetal head is permissible. If this does not result in delivery of the shoulders, the McRoberts maneuver has been universally recognized as a safe and effective procedure for allowing the infants anterior shoulder to be freed. Other maneuvers that are more invasive and carry higher risks should be used only if the above maneuvers are ineffective. Applying fundal pressure without other maneuvers has been shown to cause a 77% complication rate and should be avoided.
Ref: Cunningham FG, Gant NF, Leveno KJ, et al: Williams Obstetrics, ed 21. McGraw-Hill, 2001, pp 459-464.

Item 69 ANSWER: C

Osteitis pubis is characterized by the chronic occurrence of pain in the lower abdomen and medial thighs with exercise. It produces tenderness over the pubic symphysis, and widening of the symphysis is often seen on plain radiographs. Pain from osteomyelitis is similar, but usually presents with fever and systemic symptoms, and does not cause widening of the symphysis. A stress fracture of the pubic ramus may occur in distance athletes, but the pain and tenderness are lateral, located over the posterior ramus. Inguinal hernia should not present with symphysis pain.
Ref: Morelli V, Smith V: Groin injuries in athletes. Am Fam Physician 2001;64(8):1405-1414.

Item 70 ANSWER: B

There is no known resistance to dehydration in the elderly. There is a smaller volume of dilution for drugs distributed in body fluids. This may result in higher concentrations of medications, sometimes approaching toxic levels, especially if the usual adult dosages are given. Because elderly patients have a diminished thirst response, they are prone to develop hypernatremia when water is not immediately available.
Ref: Cassel CK, Cohen HJ, Larson EB, et al (eds): Geriatric Medicine, ed 3. Springer-Verlag, 1997, pp 802-803. 2) Tallis R, Fillit H, Brocklehurst JC (eds): Brocklehursts Textbook of Geriatric Medicine and Gerontology, ed 5. Churchill Livingstone, 1998, pp 171, 929-931.

Item 71 ANSWER: D

The Advisory Committee on Immunization Practices of the Centers for Disease Control recommends that for a clean, minor wound, tetanus toxoid should be given if the patient has not had a tetanus toxoid shot within 10 years, with a total of at least 3 prior tetanus toxoid shots.

22

Ref:

Tintinalli JE, Kelen GD, Stapczynski JS (eds): Emergency Medicine: A Comprehensive Study Guide, ed 5. McGraw-Hill, 2000, p 286.

Item 72 ANSWER: A

Antiretroviral treatment has been proven to reduce vertical transmission of HIV from mother to child. Benefits have been shown for antenatal, intrapartum, and postpartum treatments. Zidovudine and nevirapine have the most well-established track record. Maternal vitamin A therapy has not proven to reduce HIV transmission. It was hoped that it would help maintain mucosal integrity. Chlorhexidine vaginal rinses in early labor have also not proven to reduce HIV transmission to the neonate. Cesarean section decreases the frequency of neonatal transmission of HIV, but with current viral suppression therapy the risk-to-benefit ratio of cesarean delivery is questionable. The immunologic benefits of breastfeeding do not balance the increased risk of HIV transmission to neonates who are breastfed by HIV-infected mothers.
Ref: Saloojee H, Violari A: HIV infection in children. BMJ 2001;323(7314):670-674.

Item 73 ANSWER: D

Fluoroquinolones such as ciprofloxacin have been shown to significantly reduce the duration and severity of travelers diarrhea when given for 13 days. Sulfacetamide is available only in a topical form for use in the eye. Penicillin and erythromycin are not effective against the most common cause of travelers diarrhea, enterotoxigenic E. coli.
Ref: Mandell GL, Bennett JE, Dolin R (eds): Mandell, Douglas, and Bennetts Principles and Practice of Infectious Diseases, ed 5. Churchill Livingstone, 2000, pp 1104-1105.

Item 74 ANSWER: B

The risk of pulmonary embolism is five times higher in pregnant women than in nonpregnant women of similar age, and venous thromboembolism is a leading cause of illness and death during pregnancy. Warfarin, which readily crosses the placenta, should be avoided throughout pregnancy. It is definitely teratogenic during the first trimester, and extensive fetal abnormalities have been associated with exposure to warfarin in any trimester. Because heparin does not cross the placenta, it is considered the safest anticoagulant to use during pregnancy. Initially, patients with venous thromboembolism during pregnancy should be managed with heparin given according to the recommendations for nonpregnant patients. These women should receive intravenous heparin for 510 days followed by subcutaneous heparin for the duration of the pregnancy. Warfarin can be given after delivery, since it is not present in breast milk.

23

The indications for placement of an inferior vena cava filter are not changed by pregnancy, and include any contraindication to anticoagulant therapy, the occurrence of heparin-induced thrombocytopenia, and recurrence of pulmonary embolism in a patient receiving adequate anticoagulant therapy. There are no data to support the use of aspirin for treatment or prophylaxis of pulmonary embolism either during or after pregnancy.
Ref: Toglia MR, Weg JG: Venous thromboembolism during pregnancy. N Engl J Med 1996;335(2):108-114.

Item 75 ANSWER: A

Asplenic and functionally asplenic patients are susceptible to overwhelming infection from encapsulated organisms, and should be immunized with pneumococcal, meningococcal, and Hib vaccines. The vaccines immunogenicity may be reduced when given after splenectomy or during chemotherapy; thus, they should be given at least 2 weeks before performing elective splenectomy or starting chemotherapy if possible.
Ref: Brigden ML: Detection, education and management of the asplenic or hyposplenic patient. Am Fam Physician 2001;63(3):499-506, 508.

Item 76 ANSWER: D

Antidepressants that inhibit serotonergic reuptake have been reported to interfere with sexual function. Bupropion is a norepinephrine and dopamine reuptake inhibitor with essentially no direct serotonergic activity. Improvement in sexual functioning has been reported when sustained-release bupropion was either substituted for other antidepressants or added to a regimen of SSRIs. Two recent studies have also shown that sustained-release bupropion was well tolerated in the treatment of sexual dysfunction in non-depressed women.
Ref: Modell JG, May RS, Katholi CR: Effect of bupropion-SR on orgasmic dysfunction in nondepressed subjects: A pilot study. J Sex Marital Ther 2000;26(3):231-240. 2) Segraves RT, Croft H, Kavoussi R, et al: Bupropion sustained release (SR) for the treatment of hypoactive sexual desire disorder (HSDD) in nondepressed women. J Sex Marital Ther 2001;27(3):303-316.

Item 77 ANSWER: E

Simple renal cysts are incidentally seen on abdominal imaging studies in over 30% of people over age 50, and are present in up to 50% in some autopsy series. No further evaluation is indicated for cysts that meet ultrasound criteria (i.e., thin-walled, homogeneous, fluid-filled). With cysts that appear to be complex, a renal CT with contrast is indicated. MRI has been shown to be statistically superior to CT in correctly

24

characterizing benign lesions, and may be helpful when results of a CT scan are equivocal. Simple cysts do not require aspiration or other treatment. Referral is indicated for symptomatic or complex cysts or solid masses.
Ref: Higgins JC, Fitzgerald JM: Evaluation of incidental renal and adrenal masses. Am Fam Physician 2001;63(2):288-294, 299.

Item 78 ANSWER: B

Scabies is usually not seen on the head, neck, palms, and soles in adults, but these areas are often affected in infants. Lindane may be hazardous to young infants because of its percutaneous absorption and potential neurotoxicity. Permethrin is at least equally effective, and because it is poorly absorbed and rapidly metabolized, its toxicity is low. Crotamiton cream and sulphur in petrolatum are safe options, but must be applied continuously over 35 days. Laundering of clothing and bedclothes is sufficient to prevent reinfestation. In fact, clothes may simply be hung outside for 34 days, because the isolated mite dies within 23 days.
Ref: Behrman RE, Kliegman RM, Jenson HB (eds): Nelson Textbook of Pediatrics, ed 16. WB Saunders Co, 2000, pp 2044-2045.

Item 79 ANSWER: C

It is generally not considered necessary to refer patients with classic polymyalgia rheumatica for a temporal artery biopsy in the absence of symptoms or signs of giant cell arteritis (e.g., headache, visual complaints, jaw claudication, fever, scalp tenderness, abnormal funduscopic exam). Corticosteroids should be started at relatively low doses (1020 mg daily) and the patient followed for what should be a rapid clinical response.
Ref: Epperly TD, Moore KE, Harrover JD: Polymyalgia rheumatica and temporal arteritis. Am Fam Physician 2000;62(4):789-796, 801. 2) Ytterberg SR: Polymyalgia rheumatica: Clinical features and management. Primary Care Case Reviews 2001;4(2):89-95.

Item 80 ANSWER: D

This patient manifests a rapid onset of preeclampsia at term. The symptoms of epigastric pain and headache categorize her preeclampsia as severe. These symptoms indicate that the process is well advanced and that convulsions are imminent. Treatment should focus on rapid control of symptoms and delivery of the infant.
Ref: Cunningham FG, Gant NF, Leveno KJ, et al: Williams Obstetrics, ed 21. McGraw-Hill, 2001, pp 569-571, 591-592.

25

Item 81 ANSWER: B

Children from the ages of 2 to 6 often believe they are somehow responsible for the death of a loved one. The emotional pain may be so intense that the child may react by denying the death, or may somehow feel that the death is reversible. If children wish to attend a funeral, or if their parents want them to, they should be accompanied by an adult who can provide comfort and support. Telling a child the loved one is asleep or that he or she went away usually creates false hopes for return, or it may foster a sleep phobia.
Ref: Behrman RE, Kliegman RM, Jenson HB (eds): Nelson Textbook of Pediatrics , ed 16. WB Saunders Co, 2000, p 109.

Item 82 ANSWER: D

Aspartame is completely hydrolyzed in the gut to methanol, aspartic acid, and phenylalanine, and is therefore contraindicated in patients with phenylketonuria.
Ref: Oski FA (ed): Principles and Practice of Pediatrics . JB Lippincott Co, 1990, pp 83-85. 2) Rudolph AM (ed): Rudolphs Pediatrics, ed 20. Appleton & Lange, 1996, pp 306-310.

Item 83 ANSWER: D

The sudden onset of severe abdominal pain, vomiting, and diarrhea in a patient with a cardiac source of emboli and evidence of a separate embolic event makes superior mesenteric artery embolization likely. In this case, evidence of a brachial artery embolus and a cardiac rhythm indicating atrial fibrillation suggest the diagnosis. Some patients may have a surprisingly normal abdominal examination in spite of severe pain. Microscopic hematuria and blood in the stool may both occur with embolization. Severe leukocytosis is present in more than two-thirds of patients with this problem. Diagnostic confirmation by angiography is recommended. Immediate embolectomy with removal of the propagated clot can then be accomplished and a decision made regarding whether or not the intestine should be resected. A second procedure may be scheduled to reevaluate intestinal viability.
Ref: Braunwald E, Fauci AS, Kasper DL, et al (eds): Harrisons Principles of Internal Medicine, ed 15. McGraw-Hill, 2001, p 1699. 2) Townsend CM Jr: Sabiston Textbook of Surgery, ed 16. WB Saunders Co, 2001, p 1399.

Item 84 ANSWER: C

Diagnostic paracentesis is recommended for patients with ascites of recent onset, as well as for those with chronic ascites who present with new clinical findings such as fever or abdominal pain. A neutrophil count >250/mL is diagnostic for peritonitis. Once peritonitis is diagnosed, antibiotic therapy should be started immediately without waiting for culture results. Bloody ascites with abnormal cytology may be seen with

26

hepatoma, but is not typical of peritonitis. The ascitic fluid pH does not become abnormal until well after the neutrophil count has risen, so it is a less reliable finding for treatment purposes. A protein level >1 g/dL is actually evidence against spontaneous bacterial peritonitis.
Ref: Braunwald E, Fauci AS, Kasper DL, et al (eds): Harrisons Principles of Internal Medicine, ed 15. McGraw-Hill, 2001, p 829. 2) Tierney LM, McPhee SJ, Papadakis MA (eds): Current Medical Diagnosis & Treatment 2002, ed 41. Lange Medical Books/McGraw-Hill, 2002, pp 594-596.

Item 85 ANSWER: B

Family physicians often see patients with seizure disorders well controlled on phenytoin. However, due to its many side effects and associated illnesses, careful monitoring is required. Screening for ataxia, which is often subtle, must be performed at each visit, even when following blood levels at regular intervals.
Ref: Victor M, Ropper AH: Adams and Victors Principles of Neurology, ed 7. McGraw-Hill, 2001, pp 356-361.

Item 86 ANSWER: C

Hepatotoxicity resulting from timed-release formulations of niacin has been reported in elderly individuals. Patients may be taking this drug without their physicians knowledge, feeling it is safe because it is a vitamin. Aspirin and vitamin C can result in gastrointestinal iron loss and anemia. The other medications listed, if used in moderation, would not be expected to alter laboratory findings.
Ref: Roe DA: Medications and nutrition in the elderly. Prim Care 1994;21(1):135-147. 2) Drug Facts and Comparisons, 1999 Edition. Facts and Comparisons, 1998, pp 21-22.

Item 87 ANSWER: D

This patient has a markedly reduced FVC with an FEV1 /FVC ratio of 1.14%. This is consistent with moderately severe pulmonary restriction. Most likely the patient has chronic interstitial restrictive lung disease. Nitrofurantoin can cause this picture, usually after continuous treatment for 6 or more months, and pulmonary function may be impaired permanently. A wide variety of additional causes have been described including noxious gases, pulmonary hypersensitivities, neoplasia, and systemic diseases (e.g., sarcoidosis). Management includes avoidance of the offending agent or treatment of the underlying condition.
Ref: Goldman L, Bennett JC (eds): Cecil Textbook of Medicine, ed 21. WB Saunders Co, 2000, pp 409-419. 2) Braunwald E, Fauci AS, Kasper DL, et al (eds): Harrisons Principles of Internal Medicine, ed 15. McGraw-Hill, 2001, pp 1499-1505. 3) Hardman JG, Limbird LE, Gilman AG (eds): Goodman & Gilmans The Pharmacological Basis of Therapeutics, ed 10. McGraw-Hill, 2001, pp 1184-1185.

27

Item 88 ANSWER: C

Acute mountain sickness (AMS) is a clinical syndrome which may affect as many as 12%67% of persons ascending to altitudes of 8000 feet or greater. While a standard definition of AMS does not exist, persons having three or more of the following symptoms may be considered to have AMS: headache, nausea, vomiting, sleep disturbance, anorexia, fatigue, or dyspnea. Gradual ascent is recommended to allow acclimatization. Measures which may help minimize symptoms include avoidance of alcohol, increased fluid intake, and a high-carbohydrate diet. Fluid restriction and diuretics should be avoided because of the diuresis associated with acclimatization, which may by itself cause dehydration. Antibiotics are of no benefit. $-Blockers would be harmful, by interfering with the physiologic responses of tachycardia and increased cardiac output at higher altitudes. Acetazolamide speeds the process of acclimatization. The drug is a carbonic anhydrase inhibitor which results in a renal bicarbonate diuresis and metabolic acidosis, thereby increasing ventilation and arterial oxygenation. The respiratory stimulation is particularly important during sleep, when it reduces the severe hypoxemia caused by periodic breathing. The drug also lowers cerebrospinal fluid volume and pressure, which may play an additional role in its therapeutic and prophylactic utility.
Ref: Tintinalli JE, Kelen GD, Stapczynski JS (eds): Emergency Medicine: A Comprehensive Study Guide, ed 5. McGraw-Hill, 2000, pp 1263-1265.

Item 89 ANSWER: D

Radon is a naturally occurring, inert, radioactive gas which is a decay product of uranium. It can seep from soil beneath homes and reach concentrations in excess of the EPA standard of 4 picocuries. The EPA and the Surgeon General have recommended that all homes be tested for radon levels. Radon abatement measures can reduce radon concentration below EPA thresholds. Epidemiologic studies implicate radon as the second leading cause of lung cancer in the U.S., responsible for up to 30,000 of the 150,000 annual cases of bronchogenic cancer. Risk is heightened by concomitant exposure to tobacco smoke.
Ref: Cohen BL: Lung cancer risk from residential radon: Meta-analysis of eight epidemiologic studies. J Natl Cancer Inst 1997;89(9):664. 2) Viera AJ: Radon and lung cancer. Am Fam Physician 2000;62(5):950-951.

Item 90 ANSWER: B

$-Adrenergic blockers, antidepressants, anticonvulsants, calcium channel blockers, NSAIDs, and serotonin antagonists are the major classes of drugs used for preventive migraine therapy. All of these medications result in about a 50% reduction in the frequency of headaches. The other drugs listed are useful for the treatment of acute migraine, but not for prevention.

28

Ref:

Noble SL, Moore KL: Drug treatment of migraine: Part II. Preventive therapy. Am Fam Physician 1997;56(9):2279-2286. 2) Goadsby PJ, Lipton RB, Ferrari MD: MigraineCurrent understanding and treatment. N Engl J Med 2002;346(4): 257-270.

Item 91 ANSWER: D

An acute onset and fluctuating course, along with an altered level of consciousness, illusions, and distractibility are consistent with delirium according to current diagnostic criteria. A normal neurologic and general physical examination, as well as memory and orientation problems, are common to both states.
Ref: Bross MH, Tatum NO: Delirium in the elderly patient. Am Fam Physician 1994;50(6):1325-1332. 2) Hazzard WR, Blass JP, Ettinger WH Jr, et al (eds): Principles of Geriatric Medicine and Gerontology, ed 4. McGraw-Hill, 1999, pp 1229-1237. 3) Rakel RE: Textbook of Family Practice, ed 6. WB Saunders Co, 2002, pp 1365-1367.

Item 92 ANSWER: A

This child has evidence of a mild microcytic anemia, which is most commonly caused by iron deficiency related to reduced dietary intake. Less common causes are thalassemia and lead poisoning. Hemolysis usually causes a normocytic anemia with an elevated reticulocyte count. Chronic liver disease and hypothyroidism result in macrocytic anemias.
Ref: Irwin JJ, Kirchner JT: Anemia in children. Am Fam Physician 2001;64(8):1379-1386.

Item 93 ANSWER: C

This patient meets the criteria for antisocial personality disorder, including age over 18, evidence of conduct disorder in childhood, a pattern of irresponsible and antisocial behavior since age 15, and absence of schizophrenia or manic episodes. Although the patient has some features of borderline personality disorder, such as unstable relationships, the persistently aggressive nature and lack of remorse are much more typical of antisocial personality. Although the boasting quality of the patient might appear somewhat grandiose, there are no other features to suggest mania. Abused child reaction formation is not a recognized diagnosis in the Diagnostic and Statistical Manual of Mental Disorders. Schizotypal personality disorder is not usually associated with such pervasive antisocial behavior and violence.
Ref: American Psychiatric Association: Diagnostic and Statistical Manual of Mental Disorders, ed 4. American Psychiatric Association, 1994, pp 645-650.

29

Item 94 ANSWER: A

In laryngotracheitis (croup), intramuscular or oral dexamethasone (0.6 mg/kg) has been shown to reduce the rate of subsequent admission after acute treatment. Nebulized budesonide is also useful, but is not yet available in the United States. This disease is usually viral, and antibiotics are not helpful. Humidified air is of unproven benefit, and mist tents separate children from caregivers and hinder observation and evaluation.
Ref: Jaffe DM: The treatment of croup with glucocorticoids. N Engl J Med 1998;339(8):553-555.

Item 95 ANSWER: C

Polycystic ovary disease is probably the result rather than the cause of amenorrhea. Polycystic ovaries are thought to be the result of anovulation, which in turn is likely due to a combination of hyperinsulinemia and insulin resistance which leads to abnormal hypothalamic secretion of Gn-RH. This results in a high level of estrogen, causing anovulation. Patients have higher levels of LH and low to low-normal levels of FSH. Diagnostic criteria include definite or probable menstrual dysfunction, clinical evidence of increased androgens, and the exclusion of congenital adrenal hyperplasia. Findings which support the diagnosis include perimenarchal onset, insulin resistance, an elevated LH/FSH ratio, and polycystic ovaries on ultrasonography. As many as 25% of normal women may have polycystic ovaries when examined by ultrasonography, but ultrasonography alone should not be the sole criterion for diagnosis. The use of metformin reduces insulin resistance and may normalize menstruation and restore ovulation.
Ref: Sulik SM, Heath CB: Menstrual Disorders. AAFP Home Study Self-Assessment monograph series, 2000, no 255, pp 16-17.

Item 96 ANSWER: A

Topical intranasal glucocorticoids are currently believed to be the most efficacious medications for the treatment of allergic rhinitis. They are far superior to oral preparations in terms of safety. Cromolyn sodium is also an effective topical agent for allergic rhinitis; however, it is more effective if started prior to the season of peak symptoms. Because of the high risk of rhinitis medicamentosa with chronic use of topical decongestants, these agents have limited usefulness in the treatment of allergic rhinitis. Some of the newer oral antihistamines have been found to be comparable in efficacy to intranasal steroids, but their use slightly increases the incidence of adverse effects and drug interactions. They are not as useful for congestion as for sneezing, pruritus, and rhinorrhea. Newer agents are relatively free of sedation. Overall, they are not as effective as topical glucocorticoids. Azelastine, an intranasal antihistamine, is effective in controlling symptoms but can cause somnolence and a bitter taste.
Ref: Azelastine nasal spray for allergic rhinitis. Med Lett Drugs Ther 1997;39:45-47. 2) Mometasone furoate nasal spray for allergic rhinitis. Med Lett Drugs Ther 1999;41:16-17.

30

Item 97 ANSWER: D

The glycosylated hemoglobin assay is rendered inaccurate by conditions affecting red blood cell survival, such as sickle cell disease or the presence of hemoglobin C.
Ref: Juneja R, Schwartz MW, Haas LB: The glycosylated hemoglobin dilemma. Patient Care 1997;31(7):229-230. 2) Braunwald E, Fauci AS, Kasper DL, et al (eds): Harrisons Principles of Internal Medicine, ed 15. McGraw-Hill, 2001, p 2129.

Item 98 ANSWER: E

The causes of galactorrhea are multiple, including intraductal papillomatosis, mammary duct ectasia, empty sella syndrome, hyperprolactinemia, hypothyroidism, and illicit drug ingestion. However, bilateral galactorrhea, or milk production, can be physiologic for up to 2 years after breastfeeding an infant. It is also more likely if there continues to be breast stimulation, such as this womans daily expression of milk.
Ref: Pena KS, Rosenfeld JA: Evaluation and treatment of galactorrhea. Am Fam Physician 2001;63(9):1763-1770.

Item 99 ANSWER: A

Dyspnea is a frequent and distressing symptom in terminally ill patients. In the absence of hypoxia, oxygen is not likely to be helpful. Opiates are the mainstay of symptomatic treatment.
Ref: Ross DD, Alexander CS: Management of common symptoms in terminally ill patients: Part II. Constipation, delirium and dyspnea. Am Fam Physician 2001;64(6):1019-1026.

Item 100 ANSWER: D

The lesions described are typical of fire ant bites and are not infected. Fire ants are aggressive and these lesions do not imply abuse or neglect of the child. The lesions should be thoroughly cleaned with soap and water. Corticosteroids should be employed in severe cases, although their maximum effect is not achieved until several hours after administration.
Ref: Tintinalli JE, Kelen GD, Stapczynski JS (eds): Emergency Medicine: A Comprehensive Study Guide, ed 5. McGraw-Hill, 2000, pp 1245-1246, 1607. 2) Rakel RE: Textbook of Family Practice, ed 6. WB Saunders Co, 2002, pp 836-837.

31

Item 101 ANSWER: E

Tacrolimus is an immunomodulator indicated for the treatment of atopic dermatitis when corticosteroids and other conventional remedies are inadvisable, ineffective, or not tolerated. It is approved for use in patients over 2 years of age.
Ref: Topical tacrolimus for treatment of atopic dermatitis. Med Lett Drugs Ther 2001;43:33-34. 2) From the Food and Drug Administration. JAMA 2001;285(6):724.

Item 102 ANSWER: D

Essential tremor is the most likely cause of a disabling action tremor in this age group. A resting tremor, rigidity, and other associated problems are seen with Parkinsons disease. Propranolol and primidone are the agents of choice. Alprazolam may have beneficial effects, but it is not a first-line agent because of the risk of sedation and habituation. Clonazepam has not been found to be effective in this disorder. Low doses of theophylline have been found to be somewhat beneficial, but it is considered a second-line agent for essential tremor. Selective $-blockers such as metoprolol are not as effective as propranolol.
Ref: Duthie EH Jr: Practice of Geriatrics, ed 3. WB Saunders Co, 1998, pp 339-340. 2) Goroll AH: Primary Care Medicine, ed 4. Lippincott Williams & Wilkins, 2000, pp 954-955. 3) Louis ED: Essential tremor. N Engl J Med 2001;345(12):887-891.

Item 103 ANSWER: D

While previous versions of the adult Basic Life Support guidelines recommended a ratio of 15 compressions to 2 ventilations for one-rescuer CPR and a ratio of 5 compressions to 1 ventilation for two-rescuer CPR, current evidence suggests that coronary perfusion pressure is higher after 15 uninterrupted chest compressions than it is after 5 chest compressions. Thus, the 15:2 ratio is now recommended for one or two rescuers, and applies to adult Basic Life Support provided by both laypersons and health care professionals.
Ref: ECC Guidelines, Part 3: Adult Basic Life Support. Circulation 2000;102(Suppl I):22-59.

Item 104 ANSWER: E

Pneumococcal 7-valent vaccine produces a satisfactory immune response in a 1-year-old, while polyvalent vaccine does not cause a good antibody response in children under the age of 2. Neither vaccine is available orally, and cost is not a factor. The 7-valent vaccine requires multiple doses. Neither vaccine can be combined with MMR.

32

Ref:

Preventing pneumococcal disease among infants and young children: Recommendations of the Advisory Committee on Immunization Practices. MMWR 2000;49(RR-9). 2) American Academy of Pediatrics Committee on Infectious Diseases: Technical Report: Prevention of pneumococcal infections, including the use of pneumococcal conjugate and polysaccharide vaccines and antibiotic prophylaxis. Pediatrics 2000;106(2 pt 1):367-376.

Item 105 ANSWER: D

Multiple prospective and case-control studies have shown that a moderately elevated plasma homocysteine concentration is an independent risk factor for atherothrombotic vascular disease.
Ref: Welch GN, Loscalzo J: Homocysteine and atherothrombosis. N Engl J Med 1998;338(15):1042-1050.

Item 106 ANSWER: B

At present there is no approved treatment for Raynauds disease. However, patients with this disorder reportedly experience subjective symptomatic improvement with calcium channel antagonists. Nifedipine is the calcium channel blocker of choice in patients with Raynauds disease. $-Blockers can produce arterial insufficiency of the Raynaud type, so propranolol and atenolol would be contraindicated. Drugs such as ergotamine preparations and methysergide can produce cold sensitivity, and should therefore be avoided in patients with Raynauds disease.
Ref: Goldman L, Bennett JC (eds): Cecil Textbook of Medicine, ed 21. WB Saunders Co, 2000, pp 364-365. 2) Braunwald E, Fauci AS, Kasper DL, et al (eds): Harrisons Principles of Internal Medicine, ed 15. McGraw-Hill, 2001, pp 1438-1439.

Item 107 ANSWER: A

This situation illustrates a common problem in the behavioral management of children, namely positive reinforcement or reward for a negative or undesirable behavior. By giving the child a cookie, the mother is actually increasing the likelihood that the child will whine. Putting a child in time-out for whining would be an example of negative reinforcement. Letting the child continue to whine by ignoring the behavior is termed extinction. Bonding is not a term used in behavioral management, but it describes the affectional relationship between parents and infants. Bonding occurs rapidly and shortly after birth and reflects the feelings of the parents toward a newborn (unidirectional).
Ref: Behrman RE, Kliegman RM, Jenson HB (eds): Nelson Textbook of Pediatrics , ed 16. WB Saunders Co, 2000, pp 15-16.

33

Item 108 ANSWER: D

The majority of psoriasis patients can be managed with topical agents such as betamethasone dipropionate. Systemic treatment is reserved for patients with disabling psoriasis that does not respond to topical treatment. This would include phototherapy, methotrexate, and etretinate.
Ref: Greaves MW, Weinstein GD: Treatment of psoriasis. N Engl J Med 1995;332(9):581-588.

Item 109 ANSWER: E

Abnormal uterine bleeding is a relatively common disorder which may be due to functional disorders of the hypothalamus, pituitary, or ovary, as well as uterine lesions. However, the patient who is younger than 30 years of age will rarely be found to have a structural uterine defect. Once pregnancy, hematologic disease, and renal impairment are excluded, administration of intramuscular or oral progesterone will usually produce definitive flow and control the bleeding. No further evaluation should be necessary unless the bleeding recurs. Endometrial aspiration, dilatation and curettage, and other diagnostic procedures are appropriate for recurrent problems or for older women. Estrogen would only increase the problem, which is usually due to anovulation with prolonged estrogen secretion, producing a hypertrophic endometrium.
Ref: Berek JS, Adashi EY, Hillard PA (eds): Novaks Gynecology, ed 12. Williams & Wilkins, 1996, p 349.

Item 110 ANSWER: D

Carcinoma of the gallbladder is a rare condition, and prophylactic cholecystectomy to prevent its occurrence is therefore usually not justified. The exception is when chronic calculous cholecystitis is present. This condition is associated with a 20% incidence of gallbladder carcinoma.
Ref: Feldman M, Scharschmidt BF, Sleisenger MH (eds): Sleisenger & Fordtrans Gastrointestinal and Liver Disease, ed 6. WB Saunders Co, 1998, pp 968-969, 973. 2) Tallis R, Fillit H, Brocklehurst JC (eds): Brocklehursts Textbook of Geriatric Medicine and Gerontology, ed 5. Churchill Livingstone, 1998, p 865. 3) Braunwald E, Fauci AS, Kasper DL, et al (eds): Harrisons Principles of Internal Medicine, ed 15. McGraw-Hill, 2001, p 1781.

Item 111 ANSWER: A

This study uses a case-control design. It is an explanatory study, as are the experimental, cross-sectional, and follow-up designs. A clinical series is not an explanatory study, but rather a descriptive study design.

34

Like cross-sectional and follow-up designs, case-control is an observational design that begins with an outcome (e.g., testicular cancer) and then looks for common features (e.g., diet) among people who share that outcome. A cohort, or prospective, design begins with people who have not yet experienced an outcome; characteristics of the group to be studied are measured and catalogued, and then the researchers wait for the outcome. The cross-sectional design is an observational design that takes a population or cohort and makes a simultaneous assessment of outcomes and potential predictors. It is a slice-of-time design also referred to as a prevalence study. With the experimental design, one begins with a group free of the outcome, then divides the population into two groups, allowing and likewise limiting variables between each group to see how the outcome varies between the two groups. The clinical series is a descriptive type of study that records events, observations, and activities, but does not provide a detailed explanation for the cause of disease. This type of study often leads to the development of more in-depth explanatory studies.
Ref: Gehlbach SH: Interpreting the Medical Literature: A Clinicians Guide. Collamore Press, 1981, pp 18, 24, 30. 2) Wallace RB (ed): Maxcy-Rosenau-Last Public Health & Preventive Medicine, ed 14. Appleton & Lange, 1998, pp 18-20.

Item 112 ANSWER: A

The parents are describing classic breath-holding spells. These are a form of autonomic syncope frequently misdiagnosed as seizures. They occur in early childhood and infancy. They can be of two forms: cyanotic, as described here, and pallid. The cyanotic form usually occurs after vigorous crying, while the pallid form commonly occurs after a sudden fright or minor injury. The history of a prodrome of injury, vigorous crying, or sudden fright is key to distinguishing a breath-holding spell from a seizure. Parents can be reassured that no brain damage occurs and, in the presence of a classic history, no further workup is necessary. An EKG and chest radiograph would be indicated if the history or examination suggested cardiac syncope. Blood testing would be indicated if the history suggested orthostatic hypotension or diabetes. A head CT scan would be indicated in the evaluation of seizures.
Ref: Tintinalli JE, Kelen GD, Stapczynski JS (eds): Emergency Medicine: A Comprehensive Study Guide, ed 5. McGraw-Hill, 2000, p 872.

Item 113 ANSWER: D

Older patients with appendicitis are more likely to present without classic signs and symptoms. Elevated WBC counts, rebound tenderness, guarding, and fever are less reliably seen. As a result of delays in diagnosis, perforation is found in over 65% of elderly patients at the time of diagnosis.
Ref: Hazzard WR, Blass JP, Ettinger WH Jr, et al (eds): Principles of Geriatric Medicine and Gerontology, ed 4. McGraw-Hill, 1999, p 401.

35

Item 114 ANSWER: D

Despite widespread publicity and legal wrangling, a large meta-analysis of women who have had silicone breast implants has produced no evidence of any significant increase in the risk for connective tissue disease in women who have these implants.
Ref: Janowsky EC, Kupper LL, Hulka BS: Meta-analyses of the relation between silicone breast implants and the risk of connective-tissue diseases. N Engl J Med 2000;342(11):781-790.

Item 115 ANSWER: E

Sneezing, rhinorrhea, and dry mucous membranes are common problems associated with the use of continuous positive airway pressure (CPAP) for obstructive sleep apnea. However, the major problem with CPAP is lack of compliance; patients complain of discomfort, claustrophobia, panic attacks, intrusiveness, difficulty keeping the appliance in place, and noise from the machine.
Ref: Victor LD: Obstructive sleep apnea. Am Fam Physician 1999;60(8):2279-2286.

Item 116 ANSWER: B

For patients with uncomplicated gonococcal infections of the cervix, urethra, and rectum, the Centers for Disease Control recommends treatment with cefixime, ceftriaxone, ciprofloxacin, ofloxacin, or levofloxacin. If chlamydial infection has not been ruled out, or if the likelihood of chlamydial infection is high, azithromycin or doxycycline should also be given. Because this patient is pregnant, she should not receive quinolones or tetracyclines. She should receive a cephalosporin, and ceftriaxone would be the best choice because it provides higher, more sustained levels of bactericidal activity. If chlamydial infection had not been ruled out, erythromycin or amoxicillin would also be recommended.
Ref: Sexually transmitted disease guidelines 2002. MMWR 2002;51(RR-6):36-38.

Item 117 ANSWER: D

All of these medications listed are indicated for second-line treatment of acute otitis media. Ciprofloxacin, however, is not approved for patients under age 18.
Ref: Otitis Media Guideline Panel and Consortium: Otitis media with effusion in young children. Clinical Practice Guideline. ACHPR publ no 94-0622: Agency for Health Care Policy and Research, USDHHS, 1994. 2) Behrman RE, Kliegman RM, Jenson HB (eds): Nelson Textbook of Pediatrics , ed 16. WB Saunders Co, 2000, pp 1952-1953.

36

Item 118 ANSWER: A

A young man with weight loss, oral thrush, lymphadenopathy, and ulcerative esophagitis is likely to have HIV infection. Intravenous drug use is responsible for over a quarter of HIV infections in the United States. Esophageal disease develops in more than half of all patients with advanced infection during the course of their illness. The most common pathogens causing esophageal ulceration in HIV-positive patients include Candida, herpes simplex virus, and cytomegalovirus. Identifying the causative agent through culture or tissue sampling is important for providing prompt and specific therapy.
Ref: Braunwald E, Fauci AS, Kasper DL, et al (eds): Harrisons Principles of Internal Medicine, ed 15. McGraw-Hill, 2001, pp 1884-1885. 2) Mengel MB, Schwiebert LP (eds): Ambulatory Medicine: The Primary Care of Families, ed 3. Lange Medical Books/McGraw-Hill, 2001, pp 377-381.

Item 119 ANSWER: A

The likelihood that this patient has significant thyroid disease is very low given the minimal elevation of TSH, normal clinical examination, and concomitant pneumonia. The elevated TSH level is likely due to her illness rather than to any underlying thyroid condition. Even if she had a palpable thyroid, her risk of hypothyroidism would be on the order of 5%. A TSH level >20 :U/mL in an acutely ill patient reflects true hypothyroidism only about 40% of the time. It is likely that this patient has sick euthyroid syndrome and that follow-up thyroid testing after discharge when she has recovered is appropriate and is very likely to be normal. Free T4 , rT3 , and TSH levels would be appropriate for subsequent evaluation if the patients laboratory values did not return to normal after resolution of the pneumonia.
Ref: Attia J, Margetts P, Guyatt G: Diagnosis of thyroid disease in hospitalized patients: A systematic review. Arch Intern Med 1999;159(7):658-665. 2) Braunwald E, Fauci AS, Kasper DL, et al (eds): Harrisons Principles of Internal Medicine, ed 15. McGraw-Hill, 2001, p 2075.

Item 120 ANSWER: B

According to several randomized, controlled trials, mortality rates are improved in patients with heart failure who receive $-blockers in addition to diuretics, ACE inhibitors, and occasionally, digoxin. Contraindications to $-blocker use include hemodynamic instability, heart block, bradycardia, and severe asthma. $-Blockers may be tried in patients with mild asthma or COPD as long as they are monitored for potential exacerbations. $-Blocker use has been shown to be effective in patients with NYHA Class II or III heart failure. There is no absolute threshold ejection fraction. $-Blockers have also been shown to decrease mortality in patients with a previous history of myocardial infarction, regardless of their NYHA classification.
Ref: Chavey WE II: The importance of beta blockers in the treatment of heart failure. Am Fam Physician 2000;62(11):2453-2462.

37

Item 121 ANSWER: C

At least 10 million Americans suffer from urinary incontinence. In the neurologically intact individual the most common subtypes are stress incontinence, which occurs with coughing or lifting; urge incontinence, which occurs when patients sense the urge to void but are unable to inhibit leakage long enough to reach the toilet; and overflow incontinence, when the bladder cannot empty normally and becomes overdistended. The term functional incontinence is applied to those cases where lower urinary tract function is intact but other factors such as immobility and severe cognitive impairment lead to incontinence. This patient has mild urge incontinence. The first approach to control of this problem should be behavioral. In a mild case such as this, a cure can be expected. Success rates have ranged from 30% to 90% in published studies. For more severe cases, various pharmacologic agents, including anti-cholinergics, are useful. Failure of these modalities should lead to urodynamic testing and consideration of surgery.
Ref: Busby-Whitehead J, Johnson TM: Urinary incontinence. Clin Geriatr Med 1998;14(2):285-296. 2) Hazzard WR, Blass JP, Ettinger WH Jr, et al (eds): Principles of Geriatric Medicine and Gerontology, ed 4. McGraw-Hill, 1999, pp 1597-1613.

Item 122 ANSWER: D

Of the criteria listed, the feature most critical to the diagnosis of adult attention-deficit/hyperactivity disorder (ADHD) is the presence of symptoms since childhood. An extended, consistent pattern of ADHD symptoms, dating back to childhood, should be uncovered during history taking. The recent onset of symptoms or sporadic episodes of symptoms should raise concern about the appropriateness of the diagnosis of ADHD. The other features listed are seen with ADHD, but are also seen with other psychiatric disorders. Difficulty maintaining attention and focus is seen with major depression, bipolar disorder, and substance abuse. Affective lability occurs with bipolar disorder, substance abuse, and personality disorders (e.g., borderline and antisocial personality). Impulsivity and hyperactivity are characteristic of personality disorders and bipolar disorder.
Ref: Searight HR, Burke JM, Rottnek F: Adult ADHD: Evaluation and treatment in family medicine. Am Fam Physician 2000;62(9):2077-2086, 2091-2092.

Item 123 ANSWER: A

Rabies postexposure prophylaxis (RPEP) should be given for all bat bites and most raccoon bites unless brain test results will be available within 48 hours. Bites from small rodents (e.g., rats, mice, and squirrels) never require RPEP. RPEP should be given after a domestic animal bite if it was unprovoked and/or the animal demonstrated abnormal behavior and is not available for observation.

38

Ref:

Moran GJ, Talan DA, Mower W, et al: Appropriateness of rabies postexposure prophylaxis treatment for animal exposures. JAMA 2000;284(8):1001-1007. 2) Human rabies preventionUnited States, 1999: Recommendations of the Advisory Committee on Immunization Practices (ACIP). MMWR 1999;48(RR-1):1-21.

Item 124 ANSWER: B

Symptom remission is more likely to occur after neuroleptic withdrawal in young patients than in the elderly. Tardive dyskinesia is initially exacerbated by a reduction in neuroleptic dosage, and dyskinesias decrease following an increase in the dosage. Metoclopramide has been shown to cause tardive dyskinesia with long-term treatment, and therefore would not be the best drug for the patients hiatal hernia. There is no convincing evidence that any of the traditional antipsychotic drugs is less likely to produce tardive dyskinesia than any other, but the newer atypical agents such as clozapine, risperidone, and olanzapine offer some hope for a reduced incidence.
Ref: Sadock BJ, Sadock VA (eds): Kaplan & Sadocks Comprehensive Textbook of Psychiatry, ed 7. Lippincott Williams & Wilkins, 2000, pp 295-296, 2295-2296, 3075-3076.

Item 125 ANSWER: A

Azithromycin, 1 g orally once, is effective for treatment of uncomplicated urethral or cervical infection caused by Chlamydia trachomatis. Doxycycline and amoxicillin are effective for this condition, but must be taken for 710 days. Cefixime and metronidazole are effective as single doses for uncomplicated gonorrhea and vaginal trichomoniasis respectively, but neither is indicated for the treatment of Chlamydia.
Ref: Drugs for sexually transmitted infections. Med Lett Drugs Ther 1999;41:85-90.

Item 126 ANSWER: A

Ultrasonography of the scrotum will distinguish intra- from extratesticular activity nearly 100% of the time. A trans-scrotal biopsy is absolutely contraindicated, since it could lead to inguinal node metastatic spread. Antibiotic trials are not indicated for intratesticular masses. $-hCG is produced by only a small number of testicular tumors. Watchful waiting is not appropriate for testicular masses, which are usually malignant. Doubling time of testicular cancers is thought to be 1030 days.
Ref: Kinkade S: Testicular cancer. Am Fam Physician 1999;59(9):2539-2544, 2549-2550.

39

Item 127 ANSWER: C

Anaerobic lung abscesses are most often found in a person predisposed to aspiration who complains of a productive cough associated with fever, anorexia, and weakness. Physical examination usually reveals poor dental hygiene, a fetid odor to the breath and sputum, rales, and pulmonary findings consistent with consolidation. Patients who have sarcoidosis usually do not have a productive cough and have bilateral physical findings. A persistent productive cough is not a striking finding in disseminated tuberculosis, which would be suggested by miliary calcifications on a chest film. The clinical presentation and physical findings are not consistent with a simple mass in the right hilum nor with a right pleural effusion.
Ref: Goldman L, Bennett JC (eds): Cecil Textbook of Medicine, ed 21. WB Saunders Co, 2000, pp 439-441. 2) Braunwald E, Fauci AS, Kasper DL, et al (eds): Harrisons Principles of Internal Medicine, ed 15. McGraw-Hill, 2001, pp 1013-1014, 1478.

Item 128 ANSWER: A

Some gynecomastia occurs in 30%50% of pubertal males. It typically appears between 12 and 15 years of age, at Tanner stages 2 and 3. Physiologic gynecomastia is usually mild and may be either unilateral or bilateral. Breast tenderness is common and also mild. The enlargement typically regresses within a few months to 2 years. Serum hormone levels are the same as in boys without gynecomastia, although the ratio of testosterone to estrogen is lower than in males at similar stages without the condition. There is no indication for chromosomal investigation or radiography. A biopsy should be considered only if the condition persists beyond mid-puberty.
Ref: Goroll AH, Mulley AG Jr: Primary Care Medicine, ed 4. Lippincott Williams & Wilkins, 2000, pp 623, 1236. 2) Modest GA: Textbook of Primary Care Medicine, ed 3. Mosby Inc, 2000, pp 184-187.

Item 129 ANSWER: A

Since 1999, the American Academy of Family Physicians (AAFP) has recommended routine influenza vaccination for all persons aged 50 and over. The CDC has also recommended this change from the traditional schedule of beginning routine vaccination at age 65. Recommendations regarding pneumococcal vaccination remain unchanged.
Ref: Zimmerman RK: Lowering the age for routine influenza vaccination to 50 years: AAFP leads the nation in influenza vaccine policy. Am Fam Physician 1999;60(7):2061-2070. 2) Prevention and control of influenza: Recommendations of the Advisory Committee on Immunization Practices (ACIP). MMWR 2002:51(RR03):1-31.

40

Item 130 ANSWER: E

A chronic user of cocaine, like the chronic user of alcohol, does not always fit the classic description of dependence, and the physician must therefore consider the diagnosis in all patients with episodic depression and peculiar mood swings. Organic symptoms are like those of amphetamine use, mainly hyperpyrexia, tachycardia, and even cardiac arrhythmias. Routine, continued cocaine snorting often leads to nasal mucosal congestion and occasional septal perforation. Paranoid ideation is sometimes seen with the use of cocaine and other stimulants. The patients age and normal mental status make schizophrenia unlikely. Panic disorder is not complicated by paranoid behavior.
Ref: Sadock BJ, Sadock VA (eds): Kaplan & Sadocks Comprehensive Textbook of Psychiatry, ed 7. Lippincott Williams & Wilkins, 2000, pp 1003-1004.

Item 131 ANSWER: D

Cervical neoplasia is thought to be the result of sexually acquired infection with the human papillomavirus (HPV). In the patient described, the likelihood of detecting an abnormality is minimal, and screening can therefore be safely stopped.
Ref: Sawaya GF, Brown AD, Washington AE, et al: Current approaches to cervical-cancer screening. N Engl J Med 2001;344(21):1603-1607.

Item 132 ANSWER: E

Controlled trials have shown that delaying the first breastfeeding session until more than 2 hours after delivery, test weighing to determine the amount of milk ingested, and provision of formula samples to nursing mothers all substantially decrease the proportion of women successfully nursing by the first postpartum visit. Test weighing has also been shown to be inaccurate and unreliable. Ointment containing vitamins A and D was worse than nothing in clinical trials; the main effective preventive and remedial measure for sore nipples is correct positioning of the babys mouth on the breast. New mothers should be taught the breastfeeding basics, including the fact that milk supply adjusts to the infants demands, i.e., the frequency, vigor, and duration of suckling.
Ref: Behrman RE, Kliegman RM, Jenson HB (eds): Nelson Textbook of Pediatrics, ed 16. WB Saunders Co, 2000, pp 150-154.

41

Item 133 ANSWER: C

Hereditary hemochromatosis is the most common genetic disorder in the United States. Serum transferrin is the best and most sensitive screening test. Liver biopsy, long considered the gold standard for diagnosing hemochromatosis, is far too invasive. The other tests listed lack sensitivity or selectivity.
Ref: McDonnell SM, Witte D: Hereditary hemochromatosis: Preventing chronic effects of this underdiagnosed disorder. Postgrad Med 1997;102(6):83-85, 88-91, 94.

Item 134 ANSWER: D

A recent study of foot ulcers in diabetics showed that finding palpable bone at the base of an ulcer with no intervening soft tissue had a higher positive predictive value than any type of imaging study, and correlated highly with the presence of underlying osteomyelitis. Other tests listed are less likely to be diagnostic and are much more expensive than simple probing of a wound.
Ref: Sumpio BE: Foot ulcers. N Engl J Med 2000;343(11):787-793.

Item 135 ANSWER: A

Exercise therapy for peripheral vascular disease (PVD) improves maximal treadmill walking distance and functional capacity. A rigorous exercise-training program may be as beneficial as bypass surgery and more beneficial than angioplasty. The goal LDL-cholesterol level in patients with established atherosclerotic vascular disease, including those with PVD (and all patients with diabetes mellitus) should be <100 mg/dL. Tight control of diabetes mellitus has not been shown to favorably affect PVD.
Ref: Hiatt WR: Medical treatment of peripheral arterial disease and claudication. N Engl J Med 2001;344(21):1608-1621.

Item 136 ANSWER: A

Ophthalmic $-adrenergic antagonists often have systemic side effects. $2 -Blockade can cause contraction of bronchial smooth muscle, leading to bronchospasm; respiratory failure and death due to bronchospastic complications have been reported, particularly in patients with asthma or chronic obstructive pulmonary disease.
Ref: Alward WL: Medical management of glaucoma. N Engl J Med 1998;339(18):1298-1307.

42

Item 137 ANSWER: B

This patient most likely has Clostridium difficile colitis, suggested by semiformed rather than watery stool, fecal leukocytes (not seen in viral gastroenteritis or sprue), and a hospital stay greater than 2 weeks. While this disease has traditionally been associated with antibiotic use, it is posing an increasing threat to patients in hospitals and chronic-care facilities who have not been given antibiotics. The primary sources for infection in such cases have been toilets, bedpans, floors, and the hands of hospital personnel. Prompt recognition and treatment is essential to prevent patient relapse and to minimize intramural epidemics. The diarrhea of ulcerative colitis usually contains blood and occurs intermittently over a protracted course. Digoxin toxicity is likely to be accompanied by electrocardiographic and laboratory abnormalities, particularly hyper- or hypokalemia.
Ref: Dambro MR(ed): Griffiths 5 Minute Clinical Consult. Williams & Wilkins, 1998, pp 186-187, 318-319, 1110-1111, 1190.

Item 138 ANSWER: B

Injectable medroxyprogesterone acetate is not causally linked with thromboembolic events. The most common side effect is menstrual irregularities; weight gain is also a bothersome side effect. There may be a decrease in HDL-cholesterol and an increase in LDL. The cost is similar to that of combination oral contraceptives.
Ref: Earl DT, David DJ: Depo-Provera: An injectable contraceptive. Am Fam Physician 1994;49(4):891-894, 897-898. 2) Speroff L, Glass RH, Kase NG: Clinical Gynecologic Endocrinology and Infertility, ed 6. Lippincott Williams & Wilkins, 1999, pp 962-968.

Item 139 ANSWER: D

Bullous impetigo is a localized skin infection characterized by large bullae; it is caused by a group 2 phage type of Staphylococcus aureus. Cultures of fluid from an intact blister will reveal the causative agent. Ordinary penicillin is usually effective in streptococcal impetigo, but early treatment does not appear to lessen the occurrence of acute glomerulonephritis.
Ref: Behrman RE, Kliegman RM, Jenson HB (eds): Nelson Textbook of Pediatrics, ed 16. WB Saunders Co, 2000, pp 2028-2029.

43

Item 140 ANSWER: C

Of the responses listed, only acute, severe pelvic discomfort caused by myomas is an indication for hysterectomy according to ACOG guidelines based on evidence-based criteria. The others have been indications in the past, but current quality assurance evidence no longer supports hysterectomy for these conditions.
Ref: Kramer MG, Reiter RC: Hysterectomy: Indications, alternatives and predictors. Am Fam Physician 1997;55(3):827-834.

Item 141 ANSWER: D

The failure to provide immunizations because of perceived contraindications is one of the most common reasons for an inadequately protected population. A PPD may be falsely negative if administered 230 days after MMR administration, not the reverse. If the patient is immunodeficient or pregnant, rather than a household contact, then MMR is contraindicated. Breastfeeding is not a contraindication.
Ref: Institute for Clinical Systems Integration: Pediatric immunization. Postgrad Med 1996;100(5):213-225. 2) Pickering LK (ed): 2000 Red Book: Report of the Committee on Infectious Diseases, ed 25. American Academy of Pediatrics, 2000, pp 34-35, 38-39, 393-395.

Item 142 ANSWER: B

While all of the conditions listed are associated with flashing light before the eyes, a sudden shower of flashing lights is highly suggestive of retinal detachment. Migraine syndrome rarely begins in the eighth decade, vitreous floaters are more likely to be perceived as spots rather than lights, and central retinal vein thrombosis would likely cause more loss of vision. Emergency referral to an ophthalmologist is mandatory in cases of probable retinal detachment.
Ref: Hazzard WR, Blass JP, Ettinger WH Jr, et al (eds): Principles of Geriatric Medicine and Gerontology, ed 4. McGraw-Hill, 1999, p 612.

Item 143 ANSWER: D

According to the World Health Organization, an individual with a previous history of deep venous thrombosis should not use combination oral contraceptives. The advantages outweigh the disadvantages in nonsmokers over 40, smokers under the age of 35, and obese women. Combination contraceptives can be used with caution or special monitoring in individuals with migraine headaches without focal neurologic symptoms who are less than 35 years old.
Ref: Stewart FH, Harper CC, Ellertson CE, et al: Clinical breast and pelvic examination requirements for hormonal contraception: Current practice vs evidence. JAMA 2001;285(17):2232-2239.

44

Item 144 ANSWER: A

Total parenteral nutrition (TPN) is indicated for patients with poorly functioning gastrointestinal tracts who cannot tolerate other means of nutritional support and for those with high caloric requirements that cannot otherwise be met. Patients who cannot swallow because of an esophageal motility problem and those who are resistant to feeding can be managed with tube feedings. Peripheral alimentation, which provides fewer calories than TPN or liquid tube feedings, would be more appropriate over the short term in patients recovering from surgery.
Ref: Humes HD (ed): Kelleys Textbook of Internal Medicine, ed 4. Lippincott Williams & Wilkins, 2000, pp 1062-1064.

Item 145 ANSWER: D

Physical findings which suggest secondary hypertension include the presence of abdominal bruits, particularly those that lateralize or have a diastolic component. Excess body weight is correlated closely with increased blood pressure, but is not a cause of secondary hypertension. Hypertension is the most important risk factor for stroke, but a history of stroke is not an indication of secondary hypertension. Left ventricular hypertrophy is a result of hypertension, but is not an indication of secondary hypertension. The prevalence of hypertension is greater in African-Americans than in whites, but African-American race is not a risk factor for secondary hypertension.
Ref: The Sixth Report of the Joint National Committee on Prevention, Detection, Evaluation, and Treatment of High Blood Pressure. Arch Intern Med 1997;157(21):2413-2440.

Item 146 ANSWER: B

The treatment of foot problems in the elderly is difficult because of systemic and local infirmities, the most limiting being the poor vascular status of the foot. Conservative, supportive, and palliative therapy replaces definitive reconstructive surgical therapy. Surgical correction of a hammer toe and bunionectomy would be disastrous in an elderly patient with a small ulcer and peripheral vascular disease. The correct approach to this patient is to prescribe custom-made shoes and a protective shield with a central aperture of foam rubber placed over the hammer toe. Metatarsal pads are not useful in the treatment of hallux valgus and a rigid hammer toe.
Ref: Reichel W (ed): Care of the Elderly: Clinical Aspects of Aging, ed 4. Williams & Wilkins, 1995, p 360. 2) Hazzard WR, Blass JP, Ettinger WH Jr, et al (eds): Principles of Geriatric Medicine and Gerontology, ed 4. McGraw-Hill, 1999, pp 1556-1557.

45

Item 147 ANSWER: B

Anorexia with mild weight loss and difficulty falling asleep are common side effects of amphetamine treatment for ADHD. In most patients these side effects are tolerable and transient. While alternative drugs such as antidepressants can be substituted, these usually do not work as well and have their own potential side effects.
Ref: Szymanski ML, Zolotor A: Attention-deficit/hyperactivity disorder: Management. Am Fam Physician 2001;64(8): 1355-1362.

Item 148 ANSWER: B

This lesion fits best with the diagnosis of vulvar lichen planus, analogous to oral lichen planus. The genitals are a common site, and when mucous membranes are involved, there will be a lacy white or light gray reticulate pattern like that seen on the buccal mucosa with oral lesions. The cause of the condition is unknown, and it typically resolves spontaneously after 624 months. There is no treatment that shortens the disease, but topical corticosteroids may relieve the itching and improve the cosmetic appearance. This woman is a little too young for atrophic vaginitis and a little too old for vaginal adenosis. Contact dermatitis is a nonspecific erythema without white patches. Chronic candidiasis in an otherwise healthy woman would be expected to respond to conventional therapy.
Ref: Reeves JRT, Maibach H (eds): Clinical Dermatology Illustrated: A Regional Approach , ed 3. FA Davis Co, 1998, pp 206-211. 2) Mengel MB, Schwiebert LP (eds): Ambulatory Medicine: The Primary Care of Families, ed 3. Lange Medical Books/McGraw-Hill, 2001, pp 364-369. 3) Dambro MR (ed): Griffiths 5 Minute Clinical Consult. Lippincott Williams & Wilkins, 2002, pp 1156-1157.

Item 149 ANSWER: D

Carotid stenosis is an important cause of transient ischemic attacks and stroke. For patients with symptomatic carotid stenosis of more than 70%, the value of carotid endarterectomy has been firmly established on the basis of three major randomized trials. Both the North American Symptomatic Carotid Endarterectomy Trial (NASET) and the European Carotid Surgery Trial (ECST) showed that only seven or eight patients would need to undergo endarterectomy to prevent one stroke in a 5-year period. The trial by the Veterans Affairs Cooperative Studies Program also showed that endarterectomy was the best treatment in this situation.
Ref: Sacco RL: Extracranial carotid stenosis. N Engl J Med 2001;345(15):1113-1118.

46

Item 150 ANSWER: E

The patient has a history and symptoms very suggestive of a compartment syndrome of the anterior compartment of the left leg. A common cause of this serious problem is limb compression during prolonged recumbency related to drug or alcohol overdose. Early diagnosis and treatment are essential in order to avoid permanent, severe disability. The five Ps of compartment syndrome are pain, pallor, paresthesias, pulselessness, and paralysis. If distal pulses are reduced, then muscle necrosis has occurred and immediate surgical consultation is necessary. Duplex doppler ultrasonography and impedance plethysmography are used to evaluate for deep venous thrombosis. Given the decreased pulses, however, this diagnosis is much less likely and such testing may needlessly delay urgent surgical treatment. Intravenous antibiotics would be appropriate for cellulitis. This diagnosis is unlikely in this scenario given the absence of fever, erythema, and edema, and the presence of a diminished pulse. Rest, ice, and elevation are inappropriate treatment modalities for compartment syndrome.
Ref: Tintinalli JE, Kelen GD, Stapczynski JS (eds): Emergency Medicine: A Comprehensive Study Guide, ed 5. McGraw-Hill, 2000, pp 1838-1841.

Item 151 ANSWER: D

Somatization disorder is the most likely diagnosis in problem patients who are not depressed or anxious. These patients claim to have been in poor health all their lives, but have not had any outward signs of disease or abnormal physical findings. They must have at least 13 specific complaints lasting for at least 2 years and starting before the age of 30, with no evidence for other psychological disease such as panic attacks. Frequently seen complaints include vomiting, abdominal pain, painful extremities, shortness of breath, palpitations, amnesia, difficulty swallowing, a burning sensation in the sex organs, and painful menstruation.
Ref: American Psychiatric Association: Diagnostic and Statistical Manual of Mental Disorders , ed 4. American Psychiatric Association, 1994, pp 446-450.

Item 152 ANSWER: A

Many signs and symptoms of infection that are common in younger adults present less frequently or not at all in older adults. This patient has no obvious source of infection, and must be treated empirically. The other conditions listed must be considered, but the most common source of bacteremia in older adults is a urinary tract infection.
Ref: Mouton CP Bazaldua OV, Pierce B, et al: Common infections in older adults. Am Fam Physician 2001;63(2):257-268.

47

Item 153 ANSWER: C

The majority of patients presenting with spontaneous pneumothorax are tall, thin individuals under 40 years of age. Most do not have clinically apparent lung disease, and the chest pain is sometimes minimal at onset and may resolve within 24 hours even if untreated. Patients with small pneumothoraces involving less than 15% of the hemithorax may have a normal physical examination, although tachycardia is occasionally noted. The diagnosis is confirmed by chest radiographs. Studies of recurrence have found that an average of 30% of patients will have a recurrence within 6 months to 2 years. The treatment of an initial pneumothorax of less than 20% may be monitored if a patient has few symptoms. Follow-up should include a chest radiograph to assess stability at 2448 hours. Indications for treatment include progression, delayed expansion, or the development of symptoms. The majority of patients with spontaneous pneumothoraces, and perhaps almost all of them, will have subcutaneous bullae on a CT scan.
Ref: Sahn SA, Heffner JE: Spontaneous pneumothorax. N Engl J Med 2000;342(12):868-874.

Item 154 ANSWER: C

This patient most likely has endometriosis with chronic, cyclical pelvic pain. Since she is not interested in fertility, the next reasonable step is to induce a hormonal pseudopregnancy using combination oral contraceptives.
Ref: Olive DL, Pritts EA: Treatment of endometriosis. N Engl J Med 2001;345(4):266-275.

Item 155 ANSWER: C

The vacuum extractor has a probable advantage compared with forceps in a number of areas. These include easier application, lower maternal anesthesia requirements, and less risk of maternal soft-tissue and fetal facial injury. There is an increased incidence of cephalohematoma. Neonatal outcomes, as measured by Apgar scores and umbilical artery blood gases, have not been shown to be significantly different between forceps and vacuum deliveries. Forceps have been associated with higher rates of successful delivery in some studies, as they may represent an option for delivery when vacuum extraction has failed. Incomplete cervical dilatation is a relative contraindication to use of the vacuum extractor.
Ref: Putta LV, Spencer JP: Assisted vaginal delivery using the vacuum extractor. Am Fam Physician 2000;62(6):1316-1320.

48

Item 156 ANSWER: A

The Centers for Disease Control and Prevention (CDC) assembled a panel of national health experts to develop evidence-based guidelines for evaluating and treating adults with acute respiratory disease. In clinical screening, the most reliable predictors of streptococcal pharyngitis are the Centor criteria. These include tonsillar exudates, tender anterior cervical lymphadenopathy, absence of cough, and history of fever. The presence of three or four of these criteria has a positive predictive value of 40%60%, and the absence of three or four of these criteria has a negative predictive value of 80%. Patients with four positive criteria should be treated with antibiotics, those with three positive criteria should be tested and treated if positive, and those with 01 positive criteria should be treated with analgesics and supportive care only. This patient has only one of the Centor criteria, and according to the panel should not be tested or treated with antibiotics.
Ref: Ressel G: Principles of appropriate antibiotic use: Part IV. Acute pharyngitis. Am Fam Physician 2001;64(5):870, 875.

Item 157 ANSWER: C

Carpal tunnel syndrome (CTS) is an entrapment neuropathy of the median nerve at the wrist, producing paresthesias and weakness of the hands. The syndrome is caused by pressure on the median nerve where it and the flexor tendons of the fingers pass through the tunnel formed by the carpal bones and the transverse carpal ligament. It usually begins with a gradual onset of numbness, tingling, and pain in the hand and wrist. Symptoms are often present at night, during sleep, and when the wrists are flexed. The symptoms occur in the thumb and the index and middle fingers, and occasionally in part of the fourth finger. The fifth finger is never involved. The thenar compartment is innervated by the median nerve and may atrophy as the syndrome progresses. The hypothenar musculature is not involved. Physical signs of CTS include a positive Phalens maneuver, which is a provocation of symptoms by sustained wrist flexion. Symptoms can be precipitated by activities which require repeated flexion, pronation, and supination of the wrist, e.g., sewing, driving, operating computers and cash registers, and playing golf.
Ref: Goldman L, Bennett JC (eds): Cecil Textbook of Medicine, ed 21. WB Saunders Co, 2000, p 1559. 2) Victor M, Ropper AH: Adams and Victors Principles of Neurology, ed 7. McGraw-Hill, 2001, pp 1433-1434.

49

Item 158 ANSWER: E

Women younger than 35 years of age give birth to 70% of infants with Down syndrome. Chorionic villus sampling is performed at an earlier gestational age (1012 weeks) than early amniocentesis (1215 weeks). With trisomy 21, second-trimester maternal serum "-fetoprotein and unconjugated estriol levels are about 25% lower than normal levels, and hCG levels are approximately two times higher than normal. The likelihood of a fetus having trisomy 21 in a patient with a positive triple test is 2%. The triple test fails to detect trisomy 21 in the fetus in 10%15% of pregnant women over the age of 35, and thus should not be regarded as an equivalent alternative to amniocentesis in these women.
Ref: Newberger DS: Down syndrome: Prenatal risk assessment and diagnosis. Am Fam Physician 2000;62(4):825-838.

Item 159 ANSWER: C

Dysthymic disorder, also known as dysthymia, is a depressive disorder characterized by mild to moderate symptoms with a duration of 2 or more years. It has an insidious onset and a waxing and waning course. There is a strong association between dysthymia and other psychiatric disorders (especially major depression, personality disorders, and social phobia) and medical conditions (cerebrovascular accidents, multiple sclerosis, AIDS, premenstrual syndrome, hypothyroidism). Long-term treatment with selective serotonin reuptake inhibitors is often necessary.
Ref: Sansone RA, Sansone LA: Dysthymic disorder: The chronic depression. Am Fam Physician 1996;53(8):2588-2596. 2) Kaplan HI, Sadock BJ: Kaplan & Sadocks Synopsis of Psychiatry: Behavioral Sciences/Clinical Psychiatry, ed 8. Lippincott Williams & Wilkins, 1998, pp 574-578.

Item 160 ANSWER: C

Bilirubin levels >17 mg/dL in full-term infants are considered pathologic rather than physiologic. In one study, infants with bilirubin concentrations over 21 mg/dL at 1872 hours after birth had a 40% probability of severe hyperbilirubinemia developing later on. The American Academy of Pediatrics recommends initiating phototherapy for bilirubin levels based on the infants age: 15 mg/dL at 2548 hours, 18 mg/dL at 4972 hours, and 20 mg/dL at 72 hours or more.
Ref: Dennery PA, Seidman DS, Stevenson DK: Neonatal hyperbilirubinemia. N Engl J Med 2001;344(8):581-590.

Item 161 ANSWER: E

When endometrial sampling shows endometrial hyperplasia and atypia, the patient has an increased risk of underlying endometrial carcinoma. Therefore, surgical dilatation and curettage is indicated.

50

Ref:

Evans MP, Fleming KC, Evans JM: Hormone replacement therapy: Management of common problems. Mayo Clin Proc 1995;70(8):800-805. 2) Scott JR, Di Saia PJ, Hammond CB, et al (eds): Danforths Obstetrics and Gynecology, ed 8. Lippincott Williams & Wilkins, 1999, pp 843-845. 3) Speroff L, Glass RH, Kase NG: Clinical Gynecologic Endocrinology and Infertility, ed 6. Lippincott Williams & Wilkins, 1999, p 587.

Item 162 ANSWER: E

Syncope with exercise is a manifestation of organic heart disease in which cardiac output is fixed and does not rise (or even fall) with exertion. Syncope, commonly on exertion, is reported in up to 42% of patients with severe aortic stenosis. Vasovagal syncope is associated with unpleasant stimuli or physiologic conditions, including sights, sounds, smells, sudden pain, sustained upright posture, heat, hunger, and acute blood loss. Transient ischemic attacks are not related to exertion. Orthostatic hypotension is associated with changing from a sitting or lying position to an upright position. Atrial myxoma is associated with syncope related to changes in position, such as bending, changing from sitting to lying, or turning over in bed.
Ref: Kapoor WN: Syncope in older persons. J Am Geriatr Soc 1994;42(4):426-436. 2) Hazzard WR, Blass JP, Ettinger WH Jr, et al (eds): Principles of Geriatric Medicine and Gerontology, ed 4. McGraw-Hill, 1999, pp 1522-1534.

Item 163 ANSWER: A

This injury is commonly called jersey finger. It is a tendinous rupture, sometimes with a bony avulsion fracture of the flexor digitorum profundus tendon. The tendon usually retracts, which makes nonsurgical treatment unlikely to result in complete healing. Splinting in hyperextension will distract the two ends of the tendon and it will not heal. A cast or flexion splint is unlikely to lend itself to complete healing because the tendon ends are unlikely to reattach to each other.
Ref: Wang QC, Johnson BA: Fingertip injuries. Am Fam Physician 2001;63(10):1961-1966.

Item 164 ANSWER: A

Senna has been shown to be safe, free of significant intestinal side effects, and beneficial over the long term. Phenolphthalein and castor oil can cause malabsorption, dehydration, lipoid pneumonia, and cathartic colon. Bisacodyl suppositories used daily can cause rectal burning, and oral bisacodyl can cause hypokalemia, abdominal cramps, and vomiting. Milk of Magnesia is a saline cathartic that can cause elevated magnesium levels and dehydration, watery stools, and fecal incontinence.
Ref: Hazzard WR, Blass JP, Ettinger WH Jr, et al (eds): Principles of Geriatric Medicine and Gerontology, ed 4. McGraw-Hill, 1999, pp 1500-1503.

51

Item 165 ANSWER: D

Heparin-induced thrombocytopenia (HIT) results from the formation of heparin-dependent IgG antibody. Characteristically developing 510 days after the initiation of heparin by any route, it is manifested by a fall in the platelet count to less than 50% of the baseline or to an absolute level of <50,000/mm3 , mandating discontinuation of heparin. HIT is best envisioned as a state of excessive and unrestrained thrombin generation, and in this prothrombotic mileu warfarin treatment may intensify this state and must also be stopped. Direct thrombin inhibitors have been approved by the FDA for use as anticoagulants in patients with HIT. The primary treatment of HIT is to stop heparin and warfarin and substitute a non-heparin anticoagulant. A hypercoagulable state exists for several days after heparin is stopped, mandating continuation of monitoring and further antithrombotic measures. Although lowmolecular-weight heparins appear less likely to stimulate HIT-IgG antibody formation, they are also contraindicated, since HIT-IgG crossreacts with all lowmolecular-weight heparins.
Ref: Ballard JO: Anticoagulant-induced thrombosis. JAMA 1999;282(4):310-312. 2) Argatroban for treatment of heparin- induced thrombocytopenia. Med Lett Drugs Ther 2001;43:11-12. 3) Rakel RE, Bope ET (eds): Conns Current Therapy 2001. WB Saunders Co, 2001, pp 416-418.

Item 166 ANSWER: C

Vaginismus is a psychogenic phenomenon which produces spasms of the vaginal muscles. These spasms may produce only a slight contraction at the beginning of intercourse, but at the other extreme they can cause severe pain and contractions so strong that even a fingertip cannot be inserted into the vagina. Most women who suffer from vaginismus believe that their vagina is simply too small, but it is actually normal. True vaginal spasms cannot be produced voluntarily, and in fact many women with vaginismus are sexually responsive, are orgasmic on clitoral stimulation, and may enjoy sexual play without intercourse.
Ref: Berek JS, Adashi EY, Hillard PA (eds): Novaks Gynecology, ed 12. Williams & Wilkins, 1996, pp 288-289.

Item 167 ANSWER: C

Late-onset polyuria in patients on chronic lithium therapy may indicate the development of acquired renal tubular unresponsiveness to antidiuretic hormone (nephrogenic diabetes insipidus) and is usually reversible with discontinuation of the drug. Tardive dyskinesia is mainly associated with the use of neuroleptic agents and has not been reported in association with lithium therapy; in fact, lithium has been used experimentally in the management of tardive dyskinesia. Chronic use of lithium induces a mild, benign increase in the number of circulating polymorphonuclear leukocytes, which has led to its experimental use in the treatment of neutropenia. Hyperprolactinemia has not been reported with lithium therapy, and the cardiovascular effects of lithium include hypotension, not hypertension.
Ref: Drug Facts and Comparisons, 2001 Edition. Facts and Comparisons, 2001, pp 963-964. 2) Hardman JG, Limbird LE, Gilman AG (eds): Goodman & Gilmans The Pharmacological Basis of Therapeutics, ed 10. McGraw-Hill, 2001, pp 509-510.

52

Item 168 ANSWER: A

The indolent course of the sore described favors the diagnosis of cutaneous leishmaniasis. Neither malaria nor schistosomiasis produces such sores. The chancres of syphilis and trypanosomiasis are more fleeting in duration.
Ref: Goldman L, Bennett JC (eds): Cecil Textbook of Medicine, ed 21. WB Saunders Co, 2000, pp 1958-1963.

Item 169 ANSWER: A

This patient has a classic presentation of fibrositis-fibromyalgia syndrome. In addition to small bedtime doses of amitriptyline or cyclobenzaprine, symptoms are improved by an increase in physical fitness, stress reduction, regulation of sleep schedules, and reassurance. Opiate analgesics and sedative-hypnotics are not recommended for long-term treatment of this disorder.
Ref: Klippel JH (ed): Primer on the Rheumatic Diseases, ed 12. Arthritis Foundation, 2001, pp 188-193.

Item 170 ANSWER: A

Stroke is the third leading cause of death in the United States, and hypertension is the most consistently powerful predictor of stroke. There is strong and consistent evidence supporting the efficacy of antihypertensive therapy in reducing stroke risk. Several other interventions have been shown to lower stroke risk to a lesser extent, including statin treatment of high cholesterol, daily aspirin, and regular exercise.
Ref: Bronner LL, Kanter DS, Manson JE: Primary prevention of stroke. N Engl J Med 1995;333(21):1392-1400. 2) Goldman L, Bennett JC (eds): Cecil Textbook of Medicine, ed 21. WB Saunders Co, 2000, pp 2103-2104. 3) Braunwald E, Fauci AS, Kasper DL, et al (eds): Harrisons Principles of Internal Medicine, ed 15. McGraw-Hill, 2001, p 2383.

Item 171 ANSWER: D

Menses are often irregular in young adolescents; however, menstrual cycles usually become regular within 2 years after menarche. Secondary amenorrhea is present when a female who has been menstruating has not had a period for more than three cycle intervals, or 6 months. Once pregnancy has been ruled out, a disturbance in the hypothalamic-pituitary-ovarian axis should be investigated.

53

When the history and physical examination fail to reveal any clues to the diagnosis, the initial workup begins with a serum thyroid-stimulating hormone (TSH) level, a prolactin level, and a progestational challenge. The purpose of the progestational challenge, without the addition of any exogenous estrogen, is to demonstrate whether endogenous estrogen production and the uterus are normal. If the patient develops menstrual bleeding 27 days after completing the progesterone regimen, the diagnosis of anovulation can safely be made. Evaluation for a pituitary adenoma is reserved for patients with galactorrhea with or without an elevated prolactin level. Serum FSH and LH levels are investigated when the patient fails to have withdrawal bleeding after a progestational challenge, and to assess gonadotropin or follicular activity.
Ref: Ryan KJ, Berkowitz RS, Barbieri RL, et al (eds): Kistners Gynecology and Womens Health, ed 7. Mosby Inc, 1999, pp 44-48.

Item 172 ANSWER: B

The main risk factors for prostate cancer include a positive family history, African-American race, and age over 50. Urinary symptoms associated with benign prostatic hypertrophy are not associated with increased risk, nor is uncontrolled diabetes mellitus.
Ref: Barry MJ: Prostatespecific-antigen testing for early diagnosis of prostate cancer. N Engl J Med 2001;344(18):1373-1377.

Item 173 ANSWER: A

Resting tremor is the most common presentation of Parkinsons disease. It is rare in progressive supranuclear palsy and multiple system atrophy, and less common in drug-induced parkinsonism.
Ref: Hazzard WR, Blass JP, Ettinger WH Jr, et al (eds): Principles of Geriatric Medicine and Gerontology, ed 4. McGraw-Hill, 1999, pp 1272-1274.

Item 174 ANSWER: B

This patient has acute necrotizing ulcerative gingivitis. The onset is sudden and findings include tender, bleeding gums, malodorous breath, and a bad taste in the mouth. The disease is frequently associated with systemic findings such as fever, anorexia, malaise, tachycardia, cervical lymphadenopathy, and leukocytosis. Characteristic gingival lesions are seen, appearing as marginal punched-out, crater-like depressions of the interdental gingival papillae and gingival margin. They are covered with a gray pseudomembranous slough that is demarcated from the remainder of the gingiva and bleeds when removed.

54

These lesions may be related to a single tooth, a group of teeth, or the gingiva throughout the mouth. Management includes removing predisposing factors such as stress, fatigue, heavy smoking, and poor nutrition that can cause tissue breakdown. Mouth rinses with warm half-strength hydrogen peroxide are useful. When fever and lymphadenopathy occur, antibiotic treatment is warranted and penicillin is the drug of choice. Tetracycline and erythromycin are good alternatives.
Ref: Mandell GL, Bennett JE, Dolin R (eds): Mandell, Douglas, and Bennetts Principles and Practice of Infectious Diseases , ed 5. Churchill Livingstone, 2000, p 694.

Item 175 ANSWER: A

Torsade de pointes is a unique form of ventricular tachycardia characterized by an alternating axis of depolarization rotating around a point. It can be seen in a variety of clinical settings and has been described in association with everything from ischemic heart disease to liquid protein diets. A variety of drugs, including class IA antiarrhythmic agents, other antiarrhythmics, antidepressants, phenothiazines, erythromycin, and antihistamines, have been implicated. However, quinidine is the drug most frequently associated with this pattern. It is important to recognize this form of ventricular tachycardia in order to remove iatrogenic causes, if any, and to provide appropriate treatment. The rhythm does not always respond to the usual agents for ventricular tachycardia, but does respond to lidocaine. $-Adrenergic blockade and correction of metabolic and electrolyte abnormalities will also prevent recurrence. Phenytoin and magnesium are also useful therapeutic agents.
Ref: Braunwald E, Zipes DP, Libby P (eds): Heart Disease: A Textbook of Cardiovascular Medicine, ed 6. WB Saunders Co, 2001, p 721. 2) Braunwald E, Fauci AS, Kasper DL, et al (eds): Harrisons Principles of Internal Medicine, ed 15. McGraw-Hill, 2001, p 1304.

Item 176 ANSWER: B

A cholesteatoma, shown in this picture, is a potentially serious sequela of tympanic membrane perforation. Surgical therapy is mandatory. Antibiotics are not efficacious, and observation would only allow the condition to progress further.
Ref: Ballenger JJ, Snow JB Jr (eds): Otolaryngology: Head and Neck Surgery, ed 15. Lea & Febiger, 1996, pp 1013-1017. 2) Rakel RE: Textbook of Family Practice, ed 6. WB Saunders Co, 2002, pp 441-442.

55

Item 177 ANSWER: C

The disease described is Henoch-Schnlein purpura, a form of vasculitis seen in children. A URI precedes the illness in many cases. The purpuric rash seen mainly on the legs and buttocks is classic. The normal platelet count rules out thrombocytopenic causes for the rash and this patients lack of significant illness makes meningococcemia unlikely. Henoch-Schnlein purpura is frequently accompanied by arthritis of the knees and ankles, hematuria which can progress to renal failure, and colicky abdominal pain and melena. It rarely causes CNS bleeding.
Ref: Hoekelman RA (ed): Primary Pediatric Care, ed 3. Mosby-Year Book Inc, 1997, pp 1336-1337.

Item 178 ANSWER: D

This patient has mucormycosis with the common inciting event of diabetic ketoacidosis and physical findings of black necrotic eschar. More subtle signs include proptosis of the left eye due to retinal artery and vein involvement. Ecthyma gangrenosum looks similar to this lesion but is usually located in the anogenital or axillary region and is usually rounded. Necrobiosis lipoidica diabeticorum is usually found on the anterior and lateral surfaces of the lower legs and does not cause the gangrenous black eschar seen in this patient. Clostridial sinusitis would not be likely without prior trauma.
Ref: Mandell GL, Bennett JE, Dolin R (eds): Mandell, Douglas, and Bennetts Principles and Practice of Infectious Diseases, ed 5. Churchill Livingstone, 2000, pp 2685-2691. 2) Braunwald E, Fauci AS, Kasper DL, et al (eds): Harrisons Principles of Internal Medicine, ed 15. McGraw-Hill, 2001, p 1179.

Item 179 ANSWER: C

The condition shown is representative of a simple anterior dislocation of the lunate. The semilunar shape of the lunate bone is displaced anterior to the distal radial articular surface. Occasionally, a transnavicular fracture may occur along with this injury and is termed a trans-scaphoid perilunate fracture-dislocation. There is no evidence in the radiograph shown, however, of dislocations of the other areas mentioned.
Ref: Schwartz SI (ed): Principles of Surgery, ed 7. McGraw-Hill, 1999, p 1971.

Item 180 ANSWER: C

This patient has supraventricular tachycardia. The rhythm is rapid and regular and no P-waves are evident. The patient is hemodynamically stable with no angina or evidence of heart failure, so emergency cardioversion is not indicated, nor would attempts to insert a pacemaker for this arrhythmia be indicated at this time. Neither bretylium nor lidocaine is likely to terminate this arrhythmia. The drug of choice would be adenosine, verapamil, or a $-blocker.

56

This EKG could be interpreted as atrial flutter with 2:1 block, in which case the initial treatment in a hemodynamically stable patient would be ventricular rate control with a $-blocker, calcium antagonist, or digitalis. Immediate treatment with adenosine would not be effective, but would not be harmful and could clarify the underlying rhythm disturbance.
Ref: Braunwald E, Fauci AS, Kasper DL, et al (eds): Harrisons Principles of Internal Medicine, ed 15. McGraw-Hill, 2001, p 1299.

BOOK II
PATIENT A Options 111 ANSWERS: 1) T; 2) T; 3) T; 4) F; 5) T; 6) T; 7) F; 8) F; 9) F; 10) F; 11) F Abdominal aortic aneurysms occur in 2%8% of individuals over 60 years of age, and are more prevalent in men. An increased incidence has also been noted in individuals who smoke, and those who have a first degree relative with an abdominal aortic aneurysm or peripheral vascular disease. The incidence is also increased in those with carotid artery disease or hypertension. As the population ages, the incidence of abdominal aortic aneurysms is expected to increase. $-Mode ultrasonography is a good diagnostic method for asymptomatic aneurysms. It is available in most hospitals, is relatively inexpensive, does not require ionizing radiation, reveals details of the vessel wall and associated atherosclerotic plaques, and allows accurate measurement of the aneurysm in both longitudinal and transverse dimensions. It is accurate to within 3 mm of intraoperative measurement, with a reported sensitivity of 82%99%. CT is a precise test for imaging aortic aneurysms and is often used initially. It provides good images of the aorta, aortic lumen, and branch vessels. The major disadvantages of computed tomography include its expense, the use of ionizing radiation, and the fact that the image is degraded by motion. The specificity for plain radiographs is very low. Arteriography can be used to determine arterial lumen diameter, but arteriographic assessment of aneurysm size is unreliable because most aneurysm walls are lined with thrombus. The annual risk of rupture for aneurysms 5.05.7 cm in diameter is 6.6%, while the risk for those 7.0 cm in diameter is 19%. Current SVS-ISCS guidelines recommend elective repair of aneurysms 5.0 cm or greater in diameter. Following growth every 6 months is appropriate for asymptomatic aneurysms between 3.5 and 5.0 cm. Surgery should be delayed until the lesion reaches 7.0 cm only in very high-risk patients, including those with poor left ventricular function, nonreconstructable symptomatic coronary artery disease, or severe chronic obstructive pulmonary disease.
Ref: Santilli JD, Santilli SM: Diagnosis and treatment of abdominal aortic aneurysms. Am Fam Physician 1997;56(4):1081-1090. 2) Townsend CM Jr (ed): Sabiston Textbook of Surgery, ed 16. WB Saunders Co, 2001, p 1361.

57

PATIENT B Options 1222 ANSWERS: 12) T; 13) F; 14) F; 15) T; 16) F; 17) T; 18) F; 19) F; 20) T; 21) T; 22) F The patient described in this case has acute gouty arthritis, although other possibilities such as pseudogout and septic arthritis must be excluded. Tests that would be most helpful in establishing the diagnosis include a Grams stain and examination of the fluid for crystals, if fluid can be obtained. A radiograph would offer little information, and there is no real indication of renal problems or associated diseases in this patient. Acute attacks of gout are characteristically and specifically aborted by colchicine; this response aids in the diagnosis if synovial fluid cannot be aspirated and examined for the presence of typical urate crystals. Colchicine has an unpredictable effect in pseudogout. Concerns about toxicity, particularly with the intravenous form and in the elderly, have led to increased caution in the use of colchicine. Indomethacin and other short-acting NSAIDs such as ibuprofen and diclofenac, while less specific, are effective in the treatment of acute gout. Corticosteroids are also effective, by both intra-articular and systemic routes. Uricosurics work more slowly and are not indicated in the treatment of acute gout. Probenecid would not be indicated because it could exacerbate the acute attack of gout. Nafcillin would certainly not be indicated at this time without evidence of infection in the joint itself. Allopurinol is not useful in acute gout and may precipitate an acute gouty episode.
Ref: Braunwald E, Fauci AS, Kasper DL, et al (eds): Harrisons Principles of Internal Medicine, ed 15. McGraw-Hill, 2001, pp 1994-1995. 2) Klippel JH (ed): Primer on the Rheumatic Diseases, ed 12. Arthritis Foundation, 2001, pp 313-324.

PATIENT C Options 2335 ANSWERS: 23) F; 24) F; 25) T; 26) T; 27) F; 28) F; 29) T; 30) T; 31) F; 32) T; 33) T; 34) F; 35) F Distal ureteral calculi smaller than 4 mm have at least a 50% chance of passing spontaneously; therefore, urologic consultation is not necessary in this setting. A noncontrast spiral CT scan is now probably the imaging study of choice when available, replacing intravenous pyelography. MRI studies are not useful in the workup of a kidney stone. A metabolic workup is indicated for patients with recurrent stones. A 24-hour urine collection for sodium, calcium, uric acid, citrate, and oxalate is generally recommended, along with serum calcium, phosphorous, creatinine, and uric acid levels. A parathyroid hormone level is not indicated unless hypercalcemia is demonstrated. A 24-hour urine cystine study is not indicated since the case involves a calcium oxalate stone. Hyperuricosuria is associated with calcium stones. Treatment with allopurinol and potassium citrate can be beneficial. All stone formers can probably benefit from high fluid intake and a low-sodium diet. Nuts and leafy green vegetables should be restricted in this patient because they contain oxalate. Thiazide diuretics are indicated in certain types of hypercalciuria, but not in hyperuricosuria. Probenecid increases uric acid urine secretion, and therefore is contraindicated.

58

Ref:

Zieve PD: Handbook of Ambulatory Medicine. Williams & Wilkins, 1995, pp 218-233. 2) Tanagho EA, McAninch JW (eds): Smiths General Urology, ed 15. Lange Medical Books, 2000, pp 218-233.

PATIENT D Options 3645 ANSWERS: 36) T; 37) F; 38) F; 39) F; 40) T; 41) T; 42) F; 43) F; 44) T; 45) F Of the upper brachial plexus root injuries, Erb-Duchenne palsy is the most common. The differential diagnosis includes fractures of the clavicle or humerus. The other conditions listed cause generalized neuromuscular problems rather than isolated limb findings. Obstetric and fetal factors that may lead to brachial plexus injuries include multiparity, prolonged or augmented labor, abnormal presentations, shoulder dystocia, increased birth weight, and signs of fetal distress with low Apgar scores. In this case, the only factors present were increased birth weight and prolonged labor. Generally, the prognosis depends on the severity of injury. However, complete recovery of function is seen in over 90% of infants with brachial plexus injuries, usually by 3 months of age. Therapy should begin after the first week, using regular passive range-of-motion exercises plus hand and wrist splinting when appropriate.
Ref: Taeusch HW, Ballard RA: Averys Diseases of the Newborn, ed 7. WB Saunders Co, 1998, pp 865-866. 2) McMillan JA, DeAngelis CD, Feigin RD, et al (eds): Oskis Pediatrics: Principles and Practice, ed 3. Lippincott Williams & Wilkins, 1999, p 2122.

PATIENT E Options 4654 ANSWERS: 46) F; 47) T; 48) F; 49) T; 50) T; 51) T; 52) F; 53) F; 54) T In the patient described, endometrial tissue sampling should be done for diagnostic purposes to rule out endometrial hyperplasia or even uterine cancer. Outpatient endometrial biopsy is a straightforward and frequently used method to evaluate such patients. If this is difficult for technical reasons or because of patient intolerance, or if the results obtained are inconclusive, fractional dilatation and curettage is indicated. Endometrial hyperplasia usually results from noncyclic estrogen influence on the endometrium with inadequate or no progestin withdrawal, as seen with polycystic ovarian disease, estrogen-secreting tumors, or the use of unopposed exogenous estrogen replacement therapy. Unopposed estrogen stimulation by anovulatory ovaries during the perimenopausal years or in association with obesity may also be associated with endometrial hyperplasia. Premature ovarian failure is characterized by a lack of estrogen, and the endometrium is protected during estrogen replacement therapy by the concurrent use of progestational agents.
Ref: Ryan K, Berkowitz RS, Barbieri RL, et al: Kistners Gynecology and Womens Health, ed 7. Mosby, 1999, pp 131-132.

59

PATIENT F Options 5568 ANSWERS: 55) F; 56) T; 57) T; 58) T; 59) T; 60) F; 61) F; 62) T; 63) F; 64) T; 65) T; 66) F; 67) T; 68) F Health care workers are among those who have a higher risk of exposure to tuberculosis and may be required to undergo annual screening. A skin test is considered positive if there is erythema and induration of 10 mm or greater 4872 hours after the test is administered. For immunocompromised persons, including those with HIV infection, induration of 5 mm is considered the threshold. Persons vaccinated previously with Bacillus Calmette-Guerin (BCG), used in many parts of the world to prevent tuberculosis, present a diagnostic problem because a positive reaction may or may not indicate latent tuberculosis infection. Tuberculin reactivity after BCG usually decreases with time but may be boosted by repeated tuberculin skin tests. While it is generally impossible to differentiate a BCG reaction from a true positive, reactions of 20 mm of induration or more are thought to be caused by tuberculosis infection. Asymptomatic persons with positive reactions should have a clinical evaluation, and an anterior-posterior chest radiograph. Special radiographic studies such as lordotic views of the chest should be considered on an individual basis if there is a suspicion of active tuberculosis. Sputum examination is not generally indicated for asymptomatic persons with a negative chest film. Isoniazid (INH) does not affect serum lipids nor cause acute urinary retention, but can cause nausea, vomiting, abdominal pain, peripheral neuropathy, and drug-induced hepatitis. Most clinicians insist on monthly follow-up for clinical evaluation and liver function tests in patients on isoniazid, but new recommendations advise liver function tests only for those with a prior history of liver disease or other factors which might make drug-induced hepatitis more likely (e.g., alcohol or medication use). Patients with active liver disease should not take INH.
Ref: Targeted tuberculin testing and treatment of latent tuberculosis infection. MMWR 2000;49(RR-6):1-12, 16-17, 22-28, 37-39.

PATIENT G Options 6976 ANSWERS: 69) F; 70) F; 71) F; 72) T; 73) F; 74) T; 75) F; 76) F Vertebral compression fractures are common, especially in the elderly, with a fairly typical case presented here. In general, aside from loss of height and a resulting decrease in pulmonary vital capacity, there are few significant sequelae. Spinal cord damage does not occur, and nerve root compression is rare. Pain usually resolves in a few weeks. Treatment should be directed toward improving muscular support with exercise. Immobilization should be avoided, since this will cause increased bone loss, a condition that led to the original injury. The fracture is stable, making surgical intervention unnecessary.
Ref: Cassel CK, Cohen HJ, Larson EB, et al (eds): Geriatric Medicine, ed 3. Springer-Verlag, 1997, pp 440-441. 2) Braunwald E, Fauci AS, Kasper DL, et al (eds): Harrisons Principles of Internal Medicine, ed 15. McGraw-Hill, 2001, pp 2229-2230.

60

PATIENT H Options 7787 ANSWERS: 77) T; 78) T; 79) T; 80) F; 81) F; 82) F; 83) F; 84) F; 85) F; 86) F; 87) F Patients presenting with dizziness may have any of a large number of medical disorders. Fortunately, a thorough medical history is often sufficient to make a presumptive diagnosis. Vertigo, an abnormal sensation of movement, results from abnormal functioning of the vestibular system, either centrally or peripherally. The most common cause of vertigo in the elderly is benign positional vertigo (BPV), a peripheral form of vertigo characterized by an acute onset associated with movements of the head. Symptoms are short-lived, but recur with further movement of the head. Central causes of vertigo are usually accompanied by other neurologic findings. Lightheadedness may have a cardiovascular etiology, but often has a psychogenic cause. Hypoglycemia is a relatively uncommon cause of dizziness. The patient in this case exhibits the classic history and physical findings of BPV. In such cases, no further diagnostic testing is necessary. The patient may be treated with various maneuvers to remove debris from the semicircular canals or with medications to suppress symptoms.
Ref: Goroll AH, Mulley AG Jr (eds): Primary Care Medicine, ed 4. Lippincott Williams & Wilkins, 2000, pp 941-946.

PATIENT I Options 8897 ANSWERS: 88) T; 89) T; 90) F; 91) T; 92) T; 93) T; 94) F; 95) F; 96) F; 97) F The symptoms of low energy, poor concentration, abnormal sleep (especially early morning awakening), decreased appetite, and loss of interest or pleasure lasting longer than 6 weeks should alert the practitioner to the diagnosis of major depressive disorder. Depression often presents atypically, sometimes leading to more anger or irritability instead of sadness or depressed mood. Other symptoms that can be present are feelings of guilt or worthlessness, psychomotor retardation, and suicidality. Depression needs to be differentiated from the effects of other chronic or acute diseases and the abuse of alcohol or illicit drugs. Many medications can cause side effects which mimic depression, including steroids, sedatives, antipsychotics, and drugs with anticholinergic side effects, such as first-generation antihistamines. Appropriate treatment of depression may include a selective serotonin reuptake inhibitor such as citalopram. There is no indication for risperidone (an antipsychotic) or diazepam (a long-acting benzodiazepine). Cognitive psychotherapy has been proven to benefit depressed patients when used alone or in conjunction with an antidepressant. There is no indication in this patient for prednisone or an antibiotic such as ceftriaxone.
Ref: American Psychiatric Association: Diagnostic and Statistical Manual of Mental Disorders , ed 4. American Psychiatric Association, 1994, pp 35-41. 2) Keller MB, McCullough JP, Klein DN, et al: A comparison of nefazodone, the cognitive behavioral-analysis system of psychotherapy, and their combination for the treatment of chronic depression. N Engl J Med 2000;342(20):1462-1470.

61

PATIENT J Options 98104 ANSWERS: 98) F; 99) F; 100) T; 101) F; 102) F; 103) T; 104) T This patient was experiencing an acute myocardial infarction. Ventricular tachycardia in an unstable patient is treated with cardioversion. Unstable patients include those with symptoms (e.g., chest pain or dyspnea), hypotension, congestive heart failure, ischemia, or infarction. Tissue plasminogen activator (tPA) is a useful thrombolytic agent in patients with acute myocardial infarction. Its use is contraindicated in patients with recent internal bleeding (within 3 months); a known hemorrhagic diathesis; a history of stroke, intracranial arteriovenous malformation, neoplasm, or aneurysm; severe uncontrolled hypertension at the time of or just prior to administration (systolic blood pressure >200 mm Hg or diastolic blood pressure >110 mm Hg); recent intracranial, intraocular, or intraspinal surgery or trauma; recent trauma or traumatic CPR; or recent use of other thrombolytic agents. TPA can be started and given without a cardiac angiogram to define the coronary anatomy. American College of Cardiology/American Heart Association guidelines suggest that thrombolytic therapy is considered appropriate for up to 12 hours after the onset of symptoms.
Ref: Stein JH (ed): Internal Medicine, ed 5. Mosby Inc, 1998, pp 212-214. 2) Rakel RE (ed): Conns Current Therapy 2000. WB Saunders Co, 2000, p 277. 3) Braunwald E, Zipes DP, Libby P (eds): Heart Disease: A Textbook of Cardiovascular Medicine, ed 6. WB Saunders Co, 2001, p 1221.

PATIENT K Options 105114 ANSWERS: 105) T; 106) T; 107) T; 108) F; 109) F; 110) F; 111) F; 112) F; 113) T; 114) F The history, physical examination, CBC, erythrocyte sedimentation rate (ESR), and anteroposterior and frog lateral radiographs of the hip provide an accurate basis for the differential diagnosis of hip pain in children. If the temperature and ESR are elevated, septic hip, osteomyelitis, collagen disease, and transient synovitis must be considered, and hip aspiration and synovial fluid analysis should be performed. If the temperature and ESR are normal, possible diagnoses include transient synovitis, Legg-Calv-Perthes disease, slipped capital femoral epiphysis, osteoid osteoma, and local trauma. This child has transient synovitis of the hip, a self-limiting unilateral condition which is the most common cause of a limp in children. It is most common in boys between 2 and 12 years of age with 6 being the average age of onset. Many children have a history of recent viral infection and may be febrile. In addition to the limp, hip motion is limited in internal rotation and abduction. Radiographs, a CBC, and the ESR are usually normal, but the ESR may be elevated, in which case a normal synovial fluid analysis (clear, <200 WBC/mm3 , glucose equal to blood glucose) confirms the diagnosis. Treatment consists of rest, local application of heat, and acetaminophen.

62

With a septic hip, the child is febrile; the proximal thigh is swollen and lies in a position of flexion, abduction, and external rotation; the WBC count and ESR are elevated; and a portal of entry such as a skin abscess, otitis media, or pneumonia is present. Radiographs may show evidence of joint effusion. Joint aspiration confirms the diagnosis; the fluid is opaque and yellow to green in color and loaded with WBCs (>50,000/mm3 ), with a glucose level lower than corresponding blood levels, and bacteria on a Grams stain and culture. Intravenous antibiotics and surgical decompression together are necessary to eradicate the infection. In osteomyelitis, the patient is usually febrile, affected bone sites are very painful, and the WBC count and ESR are elevated. Blood cultures will be positive in more than half of cases. Radiographs are not often helpful, as the initial changes do not occur for 1014 days after the onset of the infection. Bone scanning may detect changes as early as the first day of the disease. Treatment consists of high doses of intravenous antibiotics. Legg-Calv-Perthes (LCP) disease and slipped capital femoral epiphysis do not generally cause fever or an elevated sedimentation rate. LCP occurs most frequently in boys between the ages of 4 and 8 years. Early in the disease, radiographs will be normal but bone scanning will show decreased uptake in the femoral head. Slipped capital femoral epiphysis occurs shortly before or during the growth spurt of puberty, thus high-risk groups include males from 1017 years of age and females from 815 years of age. The patients are generally obese, and radiographs are usually diagnostic. The only reliable treatment is surgical.
Ref: Behrman RE, Kliegman RM, Jenson HB (eds): Nelson Textbook of Pediatrics , ed 16. WB Saunders Co, 2000, pp 2077-2082.

PATIENT L Options 115123 ANSWERS: 115) F; 116) F; 117) T; 118) T; 119) T; 120) F; 121) T; 122) F; 123) F With this patients history of amenorrhea, it is important to look for evidence of psychological dysfunction or emotional stress; a family history of apparent genetic anomalies; signs of a physical problem with a focus on nutritional status; abnormal growth and development; the presence of a normal reproductive tract; and evidence of CNS disease. The initial step in the workup of the amenorrheic patient, after excluding pregnancy, begins with a TSH level, a prolactin level, and a progestational challenge. If the course of progestational medication does not produce withdrawal flow, either the target organ outflow tract is inoperative or preliminary estrogen proliferation of the endometrium has not occurred. Bleeding after a cycle of combined estrogen-progesterone indicates inadequate estrogen proliferation of the endometrium. Endometrial aspiration is not indicated. Evaluation of gonadotropin levels (FSH, LH) is the next step. Elevated FSH/LH indicates that the problem is ovarian failure, whereas depressed levels indicate hypothalamic or pituitary malfunction. Karyotyping would be indicated only if there is documented ovarian failure in a woman less than age 30.
Ref: Speroff L, Glass RH, Kase NG: Clinical Gynecologic Endocrinology and Infertility, ed 6. Lippincott Williams & Wilkins, 1999, pp 424-437.

63

PATIENT M Options 124136 ANSWERS: 124) F; 125) F; 126) F; 127) T; 128) T; 129) T; 130) F; 131) F; 132) F; 133) F; 134) F; 135) F; 136) F This patient has pneumonia, most probably due to Streptococcus pneumoniae. Pulmonary function testing is inappropriate during this acute illness; the patient is unlikely to cooperate well at this time. Bronchoscopy is not indicated in the absence of signs of localized bronchial obstruction. Blood cultures may be useful for identifying the organism, although treatment should not be delayed while awaiting results. Tracheal intubation is not needed in the absence of CO2 retention or exhaustion. In an elderly woman with a community-acquired pneumonia, the most likely organisms are S. pneumoniae and Mycoplasma pneumoniae. Legionella pneumophila is an additional possibility in view of this patients age and smoking history. The sputum Grams stain makes pneumococcus the most likely responsible organism. Gentamicin and cefixime might be useful if the patient had a gram-negative pneumonia; however, they are inadequate as single drugs because they do not cover Streptococcus pneumoniae, the most likely causative agent. It is important to know local resistance patterns before selecting an initial antibiotic. In most communities initial therapy consists of either doxycycline or a newer fluoroquinolone. This patients course, persistent hypoxia, findings on pulmonary function testing, and smoking history strongly suggest a diagnosis of chronic obstructive pulmonary disease (COPD). Home management includes continued smoking cessation, low-flow oxygen only if the PaO2 is below 60 mm Hg, hydration, and prompt institution of antibiotic therapy when infection occurs. Chronic expectorants and intermittent positive pressure breathing (IPPB) are not helpful. Corticosteroids should not be used unless there is a significant reversible component.
Ref: Tierney LM Jr, McPhee SJ, Papadakis MA (eds): Current Medical Diagnosis & Treatment, ed 38. Appleton & Lange, 1999, pp 286-287. 2) Braunwald E, Fauci AS, Kasper DL, et al (eds): Harrisons Principles of Internal Medicine, ed 15. McGraw-Hill, 2001, pp 1475-1485, 1491-1499.

PATIENT N Options 137147 ANSWERS: 137) F; 138) F; 139) T; 140) F; 141) F; 142) T; 143) T; 144) T; 145) F; 146) F; 147) F The significance of meconium passage in utero and during labor and the prevention of meconium aspiration syndrome have generated a good deal of controversy. Presently, it appears clear that thin meconium is more likely to be due to maturational causes and less likely to cause meconium aspiration syndrome than thick, particulate meconium. The other postulated causes of meconium passage are acute hypoxic stress during labor and chronic antepartum hypoxic stress. In the case described, the meconium is thick, which is thought to be more dangerous. In addition, the timing of the meconium passage is unknown, since there was no known event associated with hypoxia and meconium was present at the time of membrane rupture. If this infant has in utero meconium aspiration due to gasping or respiratory effort, it will likely develop meconium aspiration syndrome despite efforts to prevent it. If the infant has not yet aspirated, suctioning of the pharynx before delivery of the shoulders and tracheal suctioning if the infant is depressed appears to lower the incidence of meconium aspiration. Deep airway suctioning and intubation is advocated by many for all infants, and would also certainly be acceptable.

64

Immediate cesarean section, oxytocin augmentation, maternal oxygen, saline lavage of the airway, surfactant instillation, and immediate positive pressure ventilation are not indicated.
Ref: Klingner MC, Kruse J: Meconium aspiration syndrome: Pathophysiology and prevention. J Am Board Fam Pract 1999;12(6):450-466.

PATIENT O Options 148160 ANSWERS: 148) T; 149) F; 150) F; 151) F; 152) T; 153) F; 154) F; 155) T; 156) T; 157) T; 158) T; 159) T; 160) T In this patients age group, anemia is defined as a hemoglobin level <12 g/dL in women and <13 g/dL in men. Additional physical examination specifically focused on signs of renal or hepatic failure is appropriate, as is laboratory evaluation for chronic disease. In patients without evidence of underlying disease the initial evaluation should include a CBC, RBC indices, and a reticulocyte count. Serum vitamin B12 and folate levels are not appropriate at this time, but would be later in the evaluation if the reticulocyte index is <2 and the RBC indices demonstrate an MCV >100 :m3 /cell. A serum iron level is likewise not appropriate unless the initial studies suggest iron deficiency. Iron deficiency anemia and anemia of chronic disease are both accompanied by a low serum iron level. The total iron-binding capacity (TIBC) is increased in classic iron deficiency anemia and below normal in anemia of chronic disease. The serum ferritin level is the most useful test, differentiating anemia of chronic disease from iron deficiency anemia in 70% of patients. Serum methylmalonic acid and homocysteine levels are sensitive for detecting subclinical vitamin B12 deficiency, and normal levels virtually exclude vitamin B12 deficiency. Iron deficiency anemia in the elderly usually results from chronic gastrointestinal blood loss caused by NSAID-induced gastritis, ulcer, colon cancer, diverticula, or angiodysplasia. Further evaluation of the gastrointestinal tract as a possible source of bleeding is therefore indicated. Once iron therapy is begun, reticulocytosis should begin within a week. If the reticulocyte count increases but the anemia does not improve, continued blood loss must be considered. If the reticulocyte count does not increase, problems with iron absorption should be considered, and intravenous iron replacement may be appropriate.
Ref: Taylor RB (ed): Family Medicine: Principles and Practice, ed 5. Springer-Verlag, 1998, pp 1108-1115. 2) Hazzard WR, Blass JP, Ettinger WH Jr, et al (eds): Principles of Geriatric Medicine and Gerontology, ed 4. McGraw-Hill, 1999, pp 901-904. 3) Smith DL: Anemia in the elderly. Am Fam Physician 2000;62(7):1565-1572.

65

You might also like